Clinical Chemistry: Liver, Gastrointestinal, Pancreas, Biliary Tract

Chapter 5


Clinical Chemistry


Liver, Gastrointestinal, Pancreas, Biliary Tract


Anthony N. Sireci; Kalpana Deveraj; Jorge L. Sepulveda



Questions


1. A 3-day-old, ex 32-week, male infant is noted to have tarry black stools in his diaper on the third day of life. The clinical team is concerned about gastrointestinal bleeding, and an Apt-Downey test is ordered on a sample of the infant’s stool. After addition of 1% sodium hydroxide, the stool extract remains pink. Which one of the following is the best interpretation of these results?


A. Fetal hemoglobin is more resistant to denaturation by alkali, and the persistent pink color of the sample implies a maternal source of blood.


B. Fetal hemoglobin is more resistant to denaturation by alkali, and the persistent pink color of the sample implies a fetal source of blood.


C. Fetal hemoglobin is less resistant to denaturation by alkali, and the persistent pink color of the sample implies a maternal source of the blood.


D. Fetal hemoglobin is less resistant to denaturation by alkali, and the persistent pink color of the sample implies a fetal source of the blood.


E. Adult hemoglobin and fetal hemoglobin react equivalently under basic conditions, and the persistent pink color of the sample is inconclusive.


2. A 34-year-old woman presents to her primary care physician with weight loss and complaints of intermittent diarrhea and constipation. The physician’s differential diagnosis includes celiac disease. Which one of the following tests is the most sensitive and specific serum biomarker for celiac disease?


A. Anti-smooth muscle antibody (SMA).


B. Anti-endomysial antibody.


C. HLA-DQ2.


D. Total immunoglobulin A (IgA) levels.


E. Anti-thyroglobulin antibody.


3. A 54-year-old man recently underwent resection of an insulinoma located in the tail of the pancreas. At initial presentation, serum chromogranin A levels were 10 times the upper limit of the reference range at 2000 ng/mL, and the patient suffered from severe hypoglycemic episodes. After resection, chromogranin A levels dropped to within the reference range and have been normal since the surgical resection. On his 1-year postoperative visit, the patient’s chromogranin A level was found to have increased to 450 ng/mL, which is twice the upper limit of the reference range and much higher than the patient’s recent baseline. The patient denied any symptoms of tumor recurrence (e.g., hypoglycemia, flushing, or diarrhea). He admits to frequent dyspepsia, which he has been treating with an over-the-counter medication. However, he cannot recall the name of the medication. Which one of the following is the best next step in the management of this patient?


A. Repeat the chromogranin A measurement after 4 to 5 half-lives have passed since discontinuing the over-the-counter medication.


B. Send the patient for a positron emission tomography (PET) scan and magnetic resonance imaging (MRI) of the abdomen.


C. Send the patient immediately to the operating room for an additional pancreatic resection.


D. Do nothing; this change is within the normal, intraindividual variation of chromogranin A.


E. Order a serum CA 19-9 level.


4. Serum levels of pepsinogen type I below the reference range in healthy adults are used in screening at-risk populations for which one of the following diseases?


A. Chronic atrophic gastritis.


B. Gastric cancer.


C. Celiac disease.


D. Gastroesophageal reflux disease.


E. Peptic ulcer disease.


5a. A 23-year-old man of Ashkenazi Jewish descent presents to his primary care physician with a 4- to 5-month history of abdominal pain, intermittent diarrhea, and constipation. He reports a 5-pound unintentional weight loss. He denies any travel history. His physician submits blood samples for a perinuclear antineutrophil cytoplasmic antibody screen (P-ANCA) and a test for anti–Saccharomyces cerevisiae antibody (ASCA). Only the ASCA test result was positive. Which one of the following disorders is most consistent with this serologic pattern?


A. Irritable bowel syndrome.


B. Ulcerative colitis.


C. Crohn’s disease.


D. Celiac disease.


E. Sarcoid.



5b. By routine testing, this patient is also found to be anemic (hemoglobin = 11 g/dL; reference range, 13.3 to 16.2 g/dL) with a mean corpuscular volume (MCV) of 103 fL (reference range, 80 to 100 fL). Ileal involvement by the patient’s underlying disease is thought to be the cause. To confirm this hypothesis, a two-step Schilling test is performed. After oral loading with radiolabeled vitamin B12, the patient receives an intravenous bolus of unlabeled vitamin B12. Urinary measurement of radiolabeled B12 is performed. The same test is then repeated with the addition of oral intrinsic factor (IF). If the patient’s pathology involves impaired ileal absorption of vitamin B12, the results of this two-part Schilling test would show which one of the following patterns?


A. Less than 5% of the radiolabeled vitamin B12 in the urine after both steps 1 and 2.


B. Less than 5% of the radiolabeled vitamin B12 in the urine after step 1 and an increase in urinary radiolabeled vitamin B12 after step 2.


C. 15% urinary radiolabeled vitamin B12 after step 1 and 15% urinary radiolabeled vitamin B12 after step 2.


D. 15% urinary radiolabeled vitamin B12 after step 1 and a decrease to less than 5% radiolabeled vitamin B12 after step 2.


E. The Schilling test is not the appropriate test.


6. A patient with exocrine pancreas insufficiency is tested for enteric causes of malabsorption using the d-xylose test. After an overnight fast, the patient is given a dose of oral d-xylose, and his urinary levels of this analyte are monitored. Which one of the following comorbidities would affect your interpretation of the d-xylose test results?


A. Type 1 diabetes.


B. Crohn’s disease.


C. Cystic fibrosis.


D. Chronic renal insufficiency.


E. Hyperthyroidism.


7. An 8-year-old girl with cystic fibrosis (i.e., a positive sweat test and homozygous for the cystic fibrosis transmembrane regulator gene (CFTR) ΔF508 mutation) is transferring her care to a new pediatrician. The patient had a diagnosis of pancreatic insufficiency for which she has been treated with oral pancreatic enzyme replacement therapy. The physician wants to confirm the diagnosis of pancreatic insufficiency and collects a stool sample for fecal elastase testing. The stool is watery and poorly formed. Which one of the following reasons would cause the laboratory to reject this specimen?


A. Fecal elastase measurements should be performed only on adult patient samples because pediatric reference ranges do not exist.


B. Fecal elastase measurements should not be performed on patients who are receiving pancreatic replacement therapy.


C. Fecal elastase measurements should be performed only on well-formed stools.


D. Fecal elastase measurement is not indicated for the diagnosis of pancreatic insufficiency.


E. Fecal trypsin, not fecal elastase, measurement should be performed.


8. A 58-year-old man from India presents to his primary care physician with a complaint of postprandial dyspepsia. His symptoms began 6 months ago after a trip to India. The patient has been self-medicating with bismuth subsalicylate, usually needed immediately after meals. The primary care physician is highly suspicious for Helicobacter pylori infection and orders serologic testing, which comes back positive. However, a follow-up urea breath test is negative. Endoscopy with biopsy demonstrates the presence of H. pylori. Which one of the following is the most likely cause of these discrepant results?


A. Urease breath testing for H. pylori is notoriously inaccurate and should not be used in clinical testing protocols.


B. The endoscopic biopsy was contaminated with H. pylori.


C. The patient’s use of bismuth subsalicylate caused a false-negative test result.


D. The patient was infected with a non–urease-producing strain of H. pylori.


E. Urease breath testing is not useful in patients of Indian descent.


9. A patient suspected of having lactose intolerance is given a lactose tolerance test. After an overnight fast, the patient is given a 50-g oral bolus of lactose in 400 mL of water. Serum glucose levels are monitored 30, 60, and 120 minutes after ingestion. In a patient with severe lactase deficiency, which one of the following changes in serum glucose would you expect to see?


A. A large spike in glucose concentrations because the lactose is absorbed and broken down into its constituent monosaccharides in the serum by serum lactase.


B. A small increase in serum glucose caused by residual lactase activity, but most lactose will not by hydrolyzed by lactase in the intestinal brush border.


C. A small increase in serum glucose because of cross-reactivity between lactose and glucose.


D. A large decrease in serum glucose concentrations because most assays are inhibited by lactose.


E. A small decrease in serum glucose concentrations because most assays are inhibited by lactose.


10. A 65-year-old man presents to a gastroenterologist with 2 weeks of abdominal cramping and foul-smelling diarrhea that floats. He also reports a recent 10-pound unintentional weight loss after a trip in the local mountains. The doctor characterizes his diarrhea as steatorrhea based on history alone but would like to characterize the cause of the fat malabsorption as either pancreatic (pancreatic insufficiency perhaps secondary to malignancy) or enteric (e.g., ileal disease or infection). He performs a two-step C14-glycerol trioleate breath test and measures the C14 content of expired CO2 as a percentage of total CO2, with and without oral pancreatic enzyme replacement. Which one of the following testing patterns would be seen if the patient’s malabsorption is a result of pancreatic insufficiency?


A. A low percentage of C14 on the first round of testing and a relative increase in C14 percentage when C14 is coadministered with pancreatic enzymes.


B. A low percentage of C14 on the first round of testing without any change in C14 percentage after administration of pancreatic enzymes.


C. A high percentage of C14 on the first round of testing without any change in C14 percentage after administration of pancreatic enzymes.


D. A high percentage of C14 on the first round of testing with a decrease in C14 percentage after administration of pancreatic enzymes.


E. A low percentage of C14 on the first round of testing with a decrease in C14 percentage after administration of pancreatic enzymes.



11a. A 31-year-old African American man presents to his primary care physician with a 3-month history of bloating and diarrhea. The symptoms occur only after certain meals, but he cannot pinpoint any additional specific triggers. A stool sample is collected and sent for multiple studies, including an examination for ova and parasites, stool osmolality, and stool pH.
Which one of the following would be a classic finding on stool osmolality and pH studies in a patient with lactase deficiency?


A. A stool osmotic gap of less than 50 mOsm/kg (low) and pH less than 5.5.


B. A stool osmotic gap of more than 125 mOsm/kg (high) and pH greater than 5.5.


C. A stool osmotic gap of 80 mOsm/kg (normal) and pH less than 5.5.


D. A stool osmotic gap of more than 125 mOsm/kg (high) and pH of 7.


E. The stool osmotic gap is not a useful assay to discriminate between secretory and osmotic diarrhea.


11b. The patient later divulges that he had taken some antibiotics he had saved from a previous sinus infection. He thought it might help treat his diarrhea. Which one of the following stool studies ordered is most affected by coadministration of antibiotics?


A. Stool water sodium level.


B. Stool water potassium level.


C. Stool pH.


D. Stool osmotic gap.


E. Stool water chloride level.


12. A 64-year-old woman presents with the chief complaint of dyspepsia. She experiences burning discomfort after most meals. She denies the use of nonsteroidal antiinflammatory drugs or aspirin, and she has never used an antacid or a proton pump inhibitor (PPI). She has no personal or family history of pancreatic lesions, parathyroid adenoma, or pituitary adenoma. Her last meal, right before her appointment, precipitated the same symptoms. A serum gastrin level is ordered to rule out Zollinger-Ellison syndrome and is reported as 300 ng/L (reference range, < 100 ng/L). Which one of the following is the best interpretation of this test result?


A. Gastrin-secreting tumor secondary to Zollinger-Ellison syndrome.


B. Falsely elevated result caused by prolonged storage at room temperature before testing.


C. Atrophic gastritis.


D. Normally elevated postprandial serum gastrin levels.


E. Serum gastrin is not the appropriate biomarker for Zollinger-Ellison syndrome.


13. A patient with stage 4 cirrhosis, decreased synthetic function (low total protein and albumin; coagulopathy), and hepatic encephalopathy is found to have a serum sodium level of 124 mEq/L (reference range, 135 to 145 mEq/L). Which one of the following is the most likely cause of the patient’s hyponatremia?


A. Activation of the renin-aldosterone system.


B. Overhydration.


C. Assay interference by elevated bilirubin.


D. Nutritional imbalance.


E. Normal variation in serum sodium.


14a. A 56-year-old woman presents with severe abdominal pain that radiates to her back. She vomited once. The symptoms began approximately 48 hours ago; she has had minimal oral intake since then. Her physical examination shows dry mucous membranes, normal bowel sounds, mild abdominal guarding, but no rebound tenderness or tympany. Her past medical history includes hypertension; hypercholesterolemia, which is well controlled with a statin; and gallstones treated electively by cholecystectomy several years ago. The patient does not have a history of weight loss or alcohol abuse. She is sent by her primary care physician to the local emergency department for a computed tomography (CT) scan of the abdomen for suspected acute pancreatitis. The CT scan findings are consistent with acute pancreatitis, with possible necrosis, and no masses are observed. Laboratory testing shows a white blood cell count of 19.2 × 109/L (reference range, 4.5 to 11.0 × 109/L), bicarbonate 12 mmol/L (reference range, 21 to 28 mmol/L), blood urea nitrogen 41 mg/dL (reference range, 10 to 20 mg/dL), calcium 7.2 mg/dL (reference range, 9.0 to 10.5 mg/dL), glucose 276 mg/dL (reference range, 75 to 115 mg/dL), aspartate aminotransferase (AST) 310 U/L (reference range, 0 to 35 U/L), alanine aminotransferase (ALT) 280 U/L (reference range, 0 to 35 U/L), and lactate dehydrogenase (LDH) 400 U/L (reference range, 100 to 190 U/L). Pancreatic enzyme levels are as follows: amylase 172 U/L (reference range, 60 to 180 U/L) and lipase 165 U/L (reference range, 0 to 150 U/L). Cholesterol and triglyceride levels were within normal limits. The specimen was drawn into a lavender-top tube and stored at room temperature for 4 hours before analysis. Which one of the following reasons best explains the normal serum amylase despite a clear clinical diagnosis of acute pancreatitis?


A. Acute or chronic alcohol abuse.


B. Interference by statins.


C. Collection of the patient’s blood sample in heparin anticoagulant.


D. Collection of the patient’s blood sample in K2-EDTA.


E. Storage of the sample at room temperature for 4 hours.


14b. If the physician wanted to be certain that the patient did not have alcohol-induced pancreatitis, which one of the following tests would be the best to use?


A. Serum amylase.


B. Lipase/amylase ratio greater than 3.


C. Carbohydrate-deficient transferrin.


D. Total and direct bilirubin.


E. Trypsinogen-1.


15. A 21-year-old man is diagnosed with celiac sprue based on a small intestinal biopsy and positive serologic testing for serum anti-endomysial immunoglobulin G (IgG) antibodies. His physician prescribes a gluten-free diet. At a follow-up appointment, the patient states that he tries to follow the diet but is uncertain whether he is always choosing gluten-free products. In addition to providing additional nutrition education, which one of the following is the best next diagnostic step to determine diet adherence and treatment efficacy?


A. Perform a repeat small intestinal biopsy.


B. Measure serum anti-endomysial IgA antibodies.


C. Measure serum anti-endomysial IgG antibodies.


D. Measure fecal fat and fecal leukocytes.


E. Measure serum anti-gliadin IgG antibodies.



16a. A 4-year-old white boy of Scandinavian descent is referred to a children’s hospital for a history of multiple upper respiratory tract infections. He was delivered by spontaneous vaginal delivery, notable only for meconium. The patient has had difficulty feeding since birth, and his growth and weight have been below the 20th percentile for age. Currently, the patient has swelling of the face and lower extremities and frothy urine. The referring outside hospital performed a chloride sweat test, which was 20 mmol/L (reference range, 4 to 60 mmol/L). They also reported a creatinine of 1.8 mg/dL (reference range, < 1.5 mg/dL) and an albumin of 3.0 mg/dL (reference range, 3.5 to 5.5 mg/dL). Despite the negative chloride sweat test, the pediatrician at the children’s hospital wants to rule out cystic fibrosis by genetic testing. Which one of the following factors could best explain a false-negative sweat chloride test result in this patient?


A. Focal segmental glomerulosclerosis.


B. Malnutrition.


C. Minimal change disease.


D. Glucose-6-phosphate dehydrogenase (G6PD) deficiency.


E. Previous fludrocortisone administration.


16b. The physician recommends molecular testing for cystic fibrosis for this child. The patient’s mother is well educated about cystic fibrosis and questions the relevance of this test because of its high cost. She is already certain that her child has cystic fibrosis and believes that a great uncle probably had the disease based on his symptoms. She asks if the test is an effective screening tool for cystic fibrosis, especially because her child is not of Ashkenazi Jewish descent. Which one of the following is the correct information that the physician could provide to the patient’s family about the sensitivity of molecular testing for cystic fibrosis?


A. The sensitivity is 80% even in African Americans.


B. The sensitivity is 75% to 90% in non-Ashkenazi white North Americans.


C. The sensitivity is below 50% in non-Ashkenazi white North Americans.


D. The chance that one of the mutations in a 25-allele–screening panel is a deletion of the codon for phenylalanine 508 (Phe-508) in CFTR is 70%.


E. Molecular testing for cystic fibrosis is a better screening strategy than the sweat chloride test.


17. A 53-year-old man has a history of multiple admissions for abdominal pain, nausea, vomiting, and dehydration, and one episode of metabolic acidosis with severe hypoxia. Acute pancreatitis was diagnosed at each admission. The patient says that he drinks a maximum of two 12-ounce beers per day, but his family relates that it is much more, which the patient is afraid to admit. The medical team is concerned about chronic pancreatitis and requests endoscopic retrograde cholangiopancreatography (ERCP). They contact the laboratory director about available laboratory tests that would support ERCP findings of chronic pancreatitis. Which one of the following combinations of ERCP results and fecal elastase-1 test results would best support the diagnosis?


A. Fecal elastase-1 of less than 100 μg pancreatic elastase/gram of stool and an abnormal ERCP.


B. Fecal elastase-1 of more than 200 μg pancreatic elastase/gram of stool and a normal ERCP.


C. Fecal elastase-1 between 100 and 200 μg pancreatic elastase/gram of stool and normal ERCP.


D. Fecal elastase-1 of less than 100 μg pancreatic elastase/gram of stool and normal ERCP.


E. Fecal elastase-1 of greater than 200 μg pancreatic elastase/gram of stool and an abnormal ERCP.


18a. A patient is suspected of having a gastrin-secreting tumor. The physician drew the blood sample 2 days ago and stored it at 4°C. Now the physician calls to ask the laboratory where to deliver the specimen and whether overnight storage will affect the result. Which one of the following answers is the best response?


A. Gastrin levels will remain unchanged, but the sample should be analyzed immediately.


B. Gastrin levels remain at 95% of their physiologic levels, and the sample is stable for another 5 days.


C. Gastrin levels will be decreased by about 50%, and the test results will be inaccurate.


D. Gastrin levels will be decreased by about 95%, and the test results will be inaccurate.


E. Gastrin levels will be decreased, but the sample should be analyzed immediately for accurate results.


18b. In this case, the physician says that the specimen was collected within 5 hours of the patient’s last meal. She asks the laboratory if this will affect the result. Which one of the following is the best response?


A. No, all gastrin isoform levels are unaffected by meals.


B. No, G17 and G34 gastrin isoform levels, but not the G14 isoform level, are unaffected by meals.


C. Yes, the patient must fast for at least 24 hours before specimen collection.


D. Yes, the patient must fast for 8 hours before specimen collection.


E. No, high-molecular-weight gastrin isoform levels are the most stable and are unaffected by meals.


19. A 44-year-old man visits his primary care physician complaining of several weeks of diarrhea and upper abdominal discomfort. His stools are slightly loose, larger than normal, and sometimes bloody, typically with dark blood. The patient, who is overweight (body mass index [BMI] = 29 kg/m2), has been trying to lose weight and began running at least 5 miles most days of the week. He now watches his diet closely, eating lean protein only once per week, but has had difficulty decreasing his caffeine intake, ingesting 3 to 5 cups of coffee per day. His medications include a multivitamin, vitamin C, and ibuprofen (at least 800 mg) daily for running-related injuries. A family history includes an uncle with polyps and a grandfather who died from complications of colon cancer. The physician thinks that the diarrhea is stress related but is concerned about the family history. Guaiac-smear tests on three consecutive stools are performed. Fecal material is spotted onto guaiac-smear slides, which the physician asks the nurse to read while he sees more patients. The nurse accidentally forgets and reads the slides the next day. Each test was positive for fecal occult blood. A colonoscopy is performed and shows no abnormalities. Hemorrhoids are not seen. The pathology of random biopsies showed unremarkable mucosa. The patient’s abdominal discomfort worsens, particularly with meals. His complete blood cell count shows no change in hemoglobin. The guaiac-smear test is repeated, the same as previously, and with the same result. Which one of the following reasons best explains the positive guaiac-smear test result?


A. Failure to rehydrate guaiac-smear slides.


B. Ingestion of chicken.


C. Ingestion of large amounts of vitamin C.


D. Ibuprofen-induced gastroesophageal reflux disease.


E. Lack of red meat ingestion.


20. A 55-year-old man with a past medical history of hypertension, hyperlipidemia, and a benign salivary gland tumor resected 10 years previously is seen by his primary care physician for his annual examination. His only complaint is minor abdominal discomfort unrelated to eating. There is no nausea, vomiting, loss of appetite, weight loss, or change in bowel habits. His family medical history is remarkable for his father having pancreatic cancer. He does not smoke and only drinks alcohol once a month. Medications include simvastatin and aspirin daily. His physician orders several laboratory tests. Results for the complete blood cell count, basic metabolic panel, lipid studies, and liver function tests (LFTs) are within normal limits. Amylase is elevated at 275 U/L (reference range, 60 to 180 U/L), and lipase is within normal limits (reference range, 0 to 150 U/L). Urine amylase is measured and is below the reference range. CT imaging of the abdomen is performed and shows no pancreatic lesions. Type 1 macroamylasemia is suspected. Which one of the following laboratory tests would confirm type 1 macroamylasia?


A. Amylase clearance/creatinine clearance ratio (Cam/CCr).


B. Trypsinogen 2.


C. Lipase/amylase ratio.


D. Urine trypsinogen activation peptide.


E. Carbohydrate-deficient transferrin.


21. A gastroenterologist sees a 53-year-old woman with persistent diarrhea. The complete blood cell count, chem-20 (serum sodium, potassium, carbon dioxide, chloride, blood urea nitrogen, creatinine, glucose, AST, ALT, total protein, albumin, total and direct bilirubin, calcium, phosphorus, γ-glutamyl transpeptidase, alkaline phosphatase, total cholesterol, uric acid, LDH), and abdominal radiograph are unremarkable. Colonoscopy findings are consistent with laxative abuse. The patient reveals that she uses a magnesium laxative because of bouts of constipation. Which one of the following sets of laboratory test results best supports this diagnosis?


A. Stool sodium of 60 mOsm/kg, potassium of 80 mOsm/kg, and osmolality of 290 mOsm/kg.


B. Stool sodium of 10 mOsm/kg, potassium of 20 mOsm/kg, and osmolality of 290 mOsm/kg.


C. Elevated serum gastrin levels.


D. More than 5 g of stool lipids over 24 hours.


E. Presence of fecal leukocytes and blood.


22. A 63-year-old woman returns to her physician because of continued gnawing epigastric pain. She previously was treated with a PPI and antibiotics for suspected peptic ulcer disease. She discontinued the antibiotics after half the course because the pain did not improve. She is now experiencing too much discomfort and returned for further workup. A nonradioactive hydrogen breath test is performed and is negative. Which one of the following is the best next step in confirming this patient’s diagnosis?


A. Continue the PPI treatment and repeat the breath test after completing antibiotic therapy.


B. Repeat the hydrogen breath test.


C. Perform endoscopy and biopsy of the esophagus and stomach.


D. Perform a urease test on a stomach and esophageal biopsy.


E. Culture the stomach or esophageal biopsy tissue at 25°C.


23. A physician wants to confirm a diagnosis of ulcerative colitis. Which one of the following sets of results would provide the best evidence for the diagnosis of ulcerative colitis and would effectively exclude Crohn’s disease?


A. Positive serum P-ANCA and negative serum ASCA.


B. Negative serum P-ANCA and positive serum ASCA.


C. Negative serum perinuclear P-ANCA and negative serum ASCA.


D. Colon and small intestinal biopsy showing discontinuous presence of granulomas, crypt abscesses, and transmural inflammation.


E. Low levels of serum vitamin B12.


24. A 47-year-old white male farmworker presents to his physician with joint pain and a 10-pound weight loss over several years. Further questioning reveals that he has had diarrhea many days of the week for several years. The patient is also HIV positive, but his disease is well controlled with highly active antiretroviral therapy (HAART). A small intestinal biopsy reveals periodic acid-Schiff (PAS)-positive, diastase-resistant, lipid-laden macrophages in the lamina propria. Which one of the following is the best next step in this patient’s management?


A. Request microbial culture of the small intestinal tissue.


B. Discontinue abacavir therapy for HIV.


C. Order a polymerase chain reaction (PCR) test of small intestinal tissue.


D. Initiate treatment with metronidazole.


E. Measure serum anti-endomysial immunoglobulin A antibodies.


25. A 46-year-old woman with a past medical history of hypertension, Paget’s disease, and alcoholism presents with abdominal and back pain. Diagnostic radiology shows features consistent with Paget’s disease and possible gallstones. Results of routine laboratory evaluation include elevated alkaline phosphatase (ALP) and gamma-glutamyl transferase (GGT). Based on the above information, which one of the following statements is most correct?


A. The patient most likely does not have alcoholic liver disease if the ALT and AST levels are normal.


B. The patient’s 5′-nucleotidase activity would add no additional diagnostic information.


C. The patient’s ALP would show at least partial heat lability on heat fractionation.


D. The patient has an elevated Regan isoenzyme.


E. The patient is expected to have an increased indirect bilirubin.


26. A patient with stage 4 cirrhosis, decreased synthetic function (low total protein, albumin, and coagulopathy) and hepatic encephalopathy is found to have a serum sodium level of 124 mEq/L (reference range, 135 to 145 mEq/L). Which one of the following is the most likely etiology for the patient’s hyponatremia?


A. Activation of the renin-aldosterone system.


B. Overhydration.


C. Assay interference by elevated bilirubin.


D. Nutritional imbalance.


E. Normal variation in serum sodium.



27. Which one of the following best describes the pathophysiology of Dubin-Johnson syndrome?


A. Autoimmune hepatitis.


B. Bile duct obstruction.


C. Cirrhosis.


D. Defective organic anion transporter.


E. Acute viral hepatitis.


28a. A 59-year-old man with a history of pulmonary hypertension and alcohol abuse presents to the emergency department with progressively worsening shortness of breath, increasing abdominal girth, and jaundice. Examination is positive for elevated jugular venous distention, abdominal fluid wave, and 2 + pitting edema bilaterally. Scleral icterus is also noted. Neurologic examination is unremarkable. A hepatic function panel is ordered because of the scleral icterus and shows minimal elevations in AST, ALT, total bilirubin, and ALP. His total protein and albumin are normal. Which one of the following choices represents the most likely pathology explaining the abnormal LFT results?


A. Cirrhosis.


B. Hepatocellular carcinoma.


C. Passive congestion.


D. Fulminant hepatic failure.


E. Normal variation in liver function.


28b. Which one of the following cardiac markers, if elevated in this patient, would support your conclusions about the patient’s hepatic pathology diagnosed in the previous question?


A. Myoglobin.


B. Troponin T.


C. Ischemia-modified albumin.


D. Brain natriuretic peptide.


E. LDH.


29. A 28-year-old man sought the care of his primary care physician when he developed tremors and became unsteady on his feet. On further questioning, his physician learned that the patient’s friends had noted that his behavior had become increasingly erratic over the past year. Routine laboratory evaluation revealed mildly elevated AST and ALT, at which point the patient was referred to a hepatologist. The hepatologist’s subsequent workup revealed serum and urine chemistry abnormalities as well as fibrosis and increased quantitative copper in liver biopsy material, resulting in the patient being treated with penicillamine. Which one of the following combinations of laboratory test results would be most consistent with diagnosis?


A. Increased serum ceruloplasmin, increased serum total copper, decreased 24-hour urine copper.


B. Decreased serum ceruloplasmin, increased serum total copper, increased 24-hour urine copper.


C. Decreased serum ceruloplasmin, decreased serum total copper, decreased 24-hour urine copper.


D. Decreased serum ceruloplasmin, decreased serum total copper, increased 24-hour urine copper.


E. Increased serum ceruloplasmin, increased serum total copper, increased 24-hour urine copper.


30. A 35-year-old obese man presents to the emergency department with epigastric pain radiating to the back and lower abdomen, nausea, and vomiting. Physical examination shows abdominal tenderness, distention, guarding, unstable blood pressure averaging 135/80 mm Hg, tachycardia, and temperature of 37.8°C. Which one of the following statements is true concerning the laboratory diagnosis of this patient with acute pancreatitis?


A. A glucose level of 60 mg/dL indicates a poor prognosis.


B. Amylase is released earlier than lipase and therefore is more sensitive for acute pancreatitis.


C. Lipase is more specific than amylase for acute pancreatitis.


D. Most commercially available amylase assays are immunoassays.


E. Elastase assays are commonly used to rule out acute pancreatitis.


31. Which one of the following tests helps distinguish the origin of an elevated serum ALP level?


A. AST.


B. ALT.


C. Anti–hepatitis B core antibody.


D. GGT.


E. Ammonia.


32. AST and ALT tests are often ordered together. Which one of the following statements is true?


A. ALT is more abundant in the liver than AST.


B. ALT is more specific than AST for liver disease.


C. The AST/ALT ratio is usually lower than 2 in severe alcoholic liver disease.


D. The AST/ALT ratio is usually lower than 2 during the acute phase of viral hepatitis.


E. AST half-life in plasma is significantly longer than that of ALT.


33. A 54-year-old male heavy smoker presented to the oncology clinic for follow-up 4 months after surgical resection of a stage III, well-differentiated adenocarcinoma of the ascending colon. After surgery, the patient was treated with adjuvant chemotherapy, including 5-fluoruracil, leucovorin, and oxaliplatin for 3 months. Preoperative plasma levels of carcinoembryonic antigen (CEA) were 5.5 ng/mL (reference range, < 5.0 ng/mL) and 3 months after initiation of chemotherapy, the levels of CEA were 9.6 ng/mL. Which one of the following statements best describes the interpretation of this test?


A. Preoperative elevation of CEA indicates a poor prognosis.


B. Increased CEA after the procedure is highly suggestive of treatment failure.


C. An early increase of more than 15% in CEA levels during chemotherapy indicates a better prognosis.


D. CEA has good accuracy for screening of early-stage (Dukes A) colon cancer.


E. Significant CEA elevations (> 10 ng/mL) are specific for colon cancer.


34a. Many laboratories use the diazo method for measuring bilirubin. Which one of the following is the end product that is measured when using this technique?


A. Unconjugated bilirubin.


B. Azobilirubin.


C. Nicotinamide adenine dinucleotide phosphate (NADP).


D. Nicotinamide adenine dinucleotide (NAD).


E. Uridine diphosphate (UDP) glucuronyltransferase.


34b. The addition of accelerants such as caffeine to the diazo method for bilirubin testing facilitates the measurement of which one of the following compounds?


A. UDP glucuronyltransferase.


B. Albumin.


C. Unconjugated bilirubin only.


D. Conjugated bilirubin only.


E. Total bilirubin.


35. A 65-year-old man with colon cancer is hospitalized with a diagnosis of sepsis caused by Clostridium perfringens. The collection of a blood sample was difficult because of poor vascular access. On centrifugation, a sample of serum appeared hemolyzed. Which one of the following tests would be least likely to be affected by in vitro hemolysis and therefore more likely to help in the diagnosis of intravascular hemolysis?


A. Plasma hemoglobin levels.


B. Plasma potassium levels.


C. Plasma AST levels.


D. Plasma LDH levels.


E. Plasma haptoglobin levels.


36. A patient has a plasma total bilirubin of 2.3 mg/dL (reference range, 0.1 to 0.4), direct bilirubin of 0.2 mg/dL (reference range, ≤ 0.2), and a urinalysis showing negative bilirubin and strongly positive urobilinogen. Which one of the following disorders is the patient most likely to have?


A. Hemolytic anemia.


B. Gilbert syndrome.


C. Crigler-Najjar syndrome.


D. Liver cirrhosis.


E. Extrahepatic cholestasis.


37. A 55-year-old man with a long history (> 10 years) of heavy drinking presents with weakness, ascites, splenomegaly, gynecomastia, and pitting edema (both feet). Which one of the following laboratory tests is most consistent with severe liver cirrhosis?


A. ALT of 10,500 U/L (reference range, < 45) and AST of 8500 U/L (reference range, < 35).


B. ALP of 650 U/L (reference range, 110 to 390) and GGT of 35 U/L (reference range, 7 to 49).


C. Albumin of 3.2 g/dL (reference range, 3.4 to 4.8) and total protein of 10.4 g/dL (reference range, 6.4 to 8.3).


D. Total bilirubin of 4.3 mg/dL (reference range, 0.2 to 1.1) and direct bilirubin of 3.2 mg/dL (reference range, < 0.2).


E. Prothrombin time of 12.2 seconds (reference range, 12.0 to 14.3) and platelet count of 350 × 109/L (reference range, 150 to 400).


38. Which one of the following is the most sensitive and specific laboratory test for primary biliary cirrhosis?


A. Total bilirubin.


B. Anti-SMAs.


C. ALP.


D. Immunoglobulin M (IgM).


E. Antimitochondrial antibodies.


39. A patient was admitted with a 6-day history of jaundice, fever, malaise, anorexia, and lower back pain. Physical examination showed icterus, a temperature of 38.8°C, heart rate of 115 beats/min, and a blood pressure of 110/60 mm Hg. Laboratory values on admission showed albumin of 4.4 g/dL (reference range, 3.5 to 4.5), ALP of 2300 U/L (reference range, 33 to 96), GGT of 345 U/L (reference range, 6 to 37), AST of 120 U/L (reference range, 12 to 38), ALT of 110 U/L (reference range, 7 to 41), total bilirubin levels of 21.2 mg/dL (reference range, 0.1 to 0.4), “direct” bilirubin levels of 18.5 mg/dL (reference range, 0.2 to 0.9), and positive urinary bilirubin. Abdominal ultrasound results were normal, but MRI findings were consistent with osteomyelitis at the left sacroiliac joint. The patient was treated with broad-spectrum antibiotics, which were switched to intravenous flucloxacillin after blood cultures grew Staphylococcus aureus. The patient recovered quickly and was discharged from the hospital 12 days after admission. Twenty days after admission, the patient was asymptomatic, physical examination was normal, and laboratory results included normal albumin, AST, ALT, ALP, and GGT. Total bilirubin was elevated at 5 mg/dL, and direct bilirubin was 4.2 mg/dL. Which one of the following is the most likely explanation for the persistent hyperbilirubinemia?


A. Gilbert syndrome.


B. Persistent intrahepatic cholestasis.


C. Persistent hemolysis.


D. Delayed clearing of albumin-bound bilirubin (delta fraction).


E. Heterophilic antibody interference with bilirubin assay.


40. A 62-year-old woman presents with fatigue and progressive abdominal distention. Her past medical history includes type 2 diabetes and hypertension. She denies smoking and drinks less than one standard alcoholic drink per day. Physical examination shows a grossly distended abdomen, without tenderness and with shifting dullness. CT reveals hepatomegaly and ascites. Echocardiography is unremarkable. Abnormal laboratory tests include hemoglobin of 9.4 g/dL (reference range, 12.5 to 16.2), MCV of 88 fL (reference range, 79 to 93), albumin of 2.9 g/dL (reference range, 3.5 to 4.5), (GGT) of 180 U/L (reference range, 7 to 64), ALP of 350 U/L (reference range, 25 to 100), and total bilirubin of 1.1 mg/dL (reference range, 0.2 to 1.1). Which one of the following autoantibodies would be most sensitive for the diagnosis of primary biliary cirrhosis?


A. Antinuclear antibodies (ANAs) with a diffuse pattern.


B. Anti-SMA.


C. ANCA.


D. Anti–pyruvate dehydrogenase (PDC-M2) antibodies.


E. Anti–glutamic acid decarboxylase antibodies (GADA).


41. An assay for total hepatitis A virus (HAV) antibodies is used as a first-line screen for the laboratory diagnosis of hepatitis A. Which one of the following is the most appropriate reflex test for a positive result?


A. No reflex test is needed.


B. Hepatitis A IgM testing.


C. Hepatitis A IgA testing.


D. Hepatitis D IgG testing.


E. Hepatitis A nucleic acid testing.


42. Which one of the following is the most appropriate test for the diagnosis of acute hepatitis B virus (HBV) infection?


A. Hepatitis B surface antigen.


B. Anti–hepatitis B core total antibodies (IgG + IgM).


C. Anti–hepatitis B surface antibodies.


D. Anti–hepatitis B envelope antibodies.


E. Hepatitis B genotyping.


43. Which one of the following is the first-line screening test for hepatitis C virus (HCV) infection?


A. Recombinant immunoblot assay (RIBA) test for HCV.


B. HCV genotyping.


C. Anti-HCV immunoglobulin M (IgM) antibodies.


D. Anti-HCV immunoglobulin G (IgG) antibodies.


E. Hepatitis C surface antigen.


Answers


1. A. Fetal hemoglobin is more resistant to denaturation by alkali, and the persistent pink color of the sample implies a maternal source of blood.
B. Fetal hemoglobin is more resistant to denaturation by alkali, and the persistent pink color of the sample implies a fetal source of blood.
C. Fetal hemoglobin is less resistant to denaturation by alkali, and the persistent pink color of the sample implies a maternal source of the blood.
D. Fetal hemoglobin is less resistant to denaturation by alkali, and the persistent pink color of the sample implies a fetal source of the blood.
Rationale for C and D: Adult hemoglobin is less resistant than fetal hemoglobin to denaturation by alkali. If the blood had been from fetal ingestion of maternal blood, the sample would turn yellow-brown because adult hemoglobin denatures under basic conditions.
E. Adult hemoglobin and fetal hemoglobin react equivalently under basic conditions, and the persistent pink color of the sample is inconclusive.
Rationale for A, B, and E: Fetal hemoglobin is more resistant than adult hemoglobin to denaturation by alkali; this forms the basis of the Apt test. The persistent pink color of the sample implies that the hemoglobin has maintained its native conformation and is likely of fetal origin. If the blood had been from fetal ingestion of maternal blood, the sample would turn yellow-brown because adult hemoglobin denatures under basic conditions.


Major points of discussion


 The source of blood in the stool of a newborn must be differentiated between an ingestion of maternal red cells during labor, an ingestion of maternal red cells during breastfeeding, and an actual gastrointestinal bleed in the newborn.


 The Apt test is performed by adding water to the stool sample, washing out the red cells, and lysing them. A 1% NaOH solution is then added to the supernatant.


 The Apt test capitalizes on the intrinsic stability of fetal hemoglobin in the presence of a base (i.e., sodium hydroxide).


 If the sample maintains its pink color, the source of the blood is most likely of fetal origin.


 If the sample turns yellow-brown, that implies that the hemoglobin is denatured and that the blood is derived from a maternal source.22


2. A. Anti–smooth muscle antibody (SMA).
Rationale: Anti-SMAs are associated with autoimmune hepatitis.
B. Anti-endomysial antibody.
Rationale: Testing for anti-endomysial immunoglobulin A (EMA) antibodies is the most sensitive and specific serologic test for celiac disease. However, these antibodies are detected by labor-intensive immunofluorescence assays that are expensive and technically difficult to perform.
C. HLA-DQ2.
Rationale: A large percentage of patients with celiac disease have the HLA-DQ2 allele (90% to 95%). However, this allele is present in approximately 30% to 40% of the general population. Therefore, this tool is not specific enough for diagnosis of this disorder.
D. Total immunoglobulin A (IgA) levels.
Rationale: Celiac disease is associated with IgA deficiency. However, rather than having diagnostic utility, this test result should be used for appropriate interpretation of negative EMA serologies. Thus, a total IgA level should always be ordered along with this specific serologic test.
E. Anti-thyroglobulin antibody.
Rationale: Although there are associations between celiac disease and other autoimmune disorders, this is not a sensitive or specific biomarker for this disorder.


Major points of discussion


 EMA autoantibody is the most sensitive and specific marker for active celiac disease.


 The target antigen for EMA is tissue transglutaminase.


 Testing for EMA is typically done by an immunofluorescence assay on tissue sections of human umbilical cord. It is technically challenging to perform and interpret.


 Because EMA is an IgA antibody and IgA deficiency is also associated with celiac disease, it is important to interpret EMA results in the context of total serum IgA levels.


 The gold standard for the diagnosis of celiac disease is endoscopic biopsy showing lymphoepithelial lesions and blunting of the small intestinal villi.


 Patients with untreated celiac disease are at higher risk for developing lymphoma.1


3. A. Repeat the chromogranin A measurement after 4 to 5 half-lives have passed since discontinuing the over-the-counter medication.
Rationale: The patient’s over-the-counter medication is likely to be a proton pump inhibitor (PPI). PPIs are known to stimulate secretion of chromogranin A and cause falsely elevated levels. The chromogranin A measurement should be repeated after most of this medication has been cleared from the circulation (~ 4 to 5 half-lives of the drug). Omeprazole is a common over-the-counter PPI.
B. Send the patient for a positron emission tomography (PET) scan and magnetic resonance imaging (MRI) of the abdomen.
Rationale: Although this is not an illogical approach, the likelihood of recurrence is low given the patient’s lack of constitutional signs and symptoms. The biochemical abnormality more likely results from a biologic false-positive test result and not true pathology.
C. Send the patient immediately to the operating room for an additional pancreatic resection.
Rationale: The patient has no signs or symptoms of a recurrent insulinoma to justify an immediate resection.
D. Do nothing; this change is within the normal, intraindividual variation of chromogranin A.
Rationale: Although there are assay-to-assay, interindividual, and intraindividual variations in chromogranin A levels, the patient has been “within normal range” for an entire year. He now presents with a twofold elevation. This is not compatible with intraindividual biologic variation.
E. Order a serum CA 19-9 level.
Rationale: This is a marker of pancreatic adenocarcinoma.


Major points of discussion


 Chromogranin A is elevated in patients with various neuroendocrine tumors, including functional and nonfunctional islet cell tumors of the pancreas; foregut, midgut, and hindgut carcinoid tumors; pheochromocytoma; medullary thyroid cancers; and neuroblastoma.


 Chromogranin A levels, in conjunction with measuring plasma serotonin and urinary 5-hydroxyindoleacetic levels, are particularly useful in the diagnosis and follow-up of carcinoid tumors.


 Chromogranin A levels are increased by the use of proton pump inhibitors and by kidney failure.


 Chromogranin A is measured by immunoassay and is, therefore, subject to interfering substances such as heterophile antibodies.


 There is no standard reference material for chromogranin A.35


4. A. Chronic atrophic gastritis.
Rationale: Serum pepsinogen levels correlate with parietal cell mass. Decreased serum pepsinogen is associated with a decrease in parietal cells, as is seen in atrophic gastritis. Chronic atrophic gastritis is a known precursor to gastric carcinoma. Screening high-risk populations, such as is done in Japan, aims to identify a group of patients with chronic atrophic gastritis who will be followed aggressively by endoscopy for the early detection of cancer.
B. Gastric cancer.
Rationale: Although serum pepsinogen levels are lower in patients with gastric carcinoma, screening programs aim to detect an at-risk population in need of more aggressive follow-up. Screening programs using serum pepsinogen are not used to diagnose patients with gastric cancer.
C. Celiac disease.
Rationale: Celiac disease involves the small intestine and is not expected to affect parietal cell mass.
D. Gastroesophageal reflux disease.
Rationale: Gastroesophageal reflux disease may result from hypersecretory states associated with increased serum pepsinogen levels.
E. Peptic ulcer disease.
Rationale: Generally, serum gastrin and secretin levels are elevated in patients with peptic ulcer disease. Parietal cell mass may theoretically be affected, but serum pepsinogen is not used to screen for this disease.


Major points of discussion


 Pepsinogens are produced in two distinct isoforms, types I and II. They are activated by enzymatic cleavage at acidic pH and degraded at alkaline pH.


 Only about 1% of secreted pepsinogens enter the circulation and can be measured in serum.


 Serum levels correlate with parietal cell mass; therefore, in states of decreased parietal cell mass, such as atrophic gastritis, pepsinogen levels are below the reference interval. High levels of pepsinogen are seen in hypersecretory states such as Zollinger-Ellison syndrome, duodenal ulcer disease, and gastrinoma.


 Severe atrophic gastritis increases the risk for the subsequent development of gastric carcinoma.


 Pepsinogen I levels in serum are measured by immunoassay and range from 20 to 107 μg/L in healthy adults.23


5a. A. Irritable bowel syndrome.
Rationale: Serologic testing is generally negative in irritable bowel syndrome.
B. Ulcerative colitis.
Rationale: Although this serologic pattern can be seen in a small percentage of patients with ulcerative colitis, these patients will more commonly test P-ANCA positive and ASCA negative.
C. Crohn’s disease.
Rationale: ASCA is positive in 65% of patients with Crohn’s disease and in 15% of patients with ulcerative colitis. P-ANCA is positive in only 20% of patients with Crohn’s disease and in 70% of patients with ulcerative colitis. Therefore, the serologic pattern of ASCA positive and P-ANCA negative is most consistent with Crohn’s disease.
D. Celiac disease.
Rationale: Celiac disease is serologically characterized by the presence of EMA autoantibodies.
E. Sarcoid.
Rationale: This disorder is characterized by elevated serum angiotensin-converting enzyme (ACE) levels.


5b. A. Less than 5% of the radiolabeled vitamin B12 in the urine after both steps 1 and 2.
Rationale: This is the classic finding in patients with ileal dysfunction caused by Crohn’s disease (as in this patient). Radiolabeled vitamin B12 is not absorbed in the ileum caused by the disease, and the addition of intrinsic factor does not correct the problem. A cutoff of less than 5% of the dosed radiolabeled vitamin B12 in the urine is generally used to define malabsorption.
B. Less than 5% of the radiolabeled vitamin B12 in the urine after step 1 and an increase in urinary radiolabeled vitamin B12 after step 2.
Rationale: This is the pattern seen in pernicious anemia. In this setting, parietal cells do not produce intrinsic factor caused by gastric pathology (e.g., atrophic gastritis, carcinoma). Intrinsic factor is required to absorb vitamin B12 in the ileum. Therefore, addition of exogenous intrinsic factor corrects the vitamin malabsorption.
C. 15% urinary radiolabeled vitamin B12 after step 1 and 15% urinary radiolabeled vitamin B12 after step 2.
Rationale: This pattern is seen in patients without vitamin B12 malabsorption.
D. 15% urinary radiolabeled vitamin B12 after step 1 and a decrease to less than 5% radiolabeled vitamin B12 after step 2.
Rationale: This is a difficult pattern to explain pathophysiologically; therefore, the test should probably be repeated.
E. The Schilling test is not the appropriate test.
Rationale: The Schilling test is helpful in differentiating ileal causes of vitamin B12 malabsorption from parietal cell causes (i.e., pernicious anemia).


Major points of discussion


 ASCA test results are positive in 65% of patients with Crohn’s disease and in only 15% of patients with ulcerative colitis.


 P-ANCA screening results are positive in 70% of patients with ulcerative colitis and in only 20% of patients with Crohn’s disease.


 Irritable bowel syndrome is serologically negative more than 95% of the time.


 Because of the diffuse nature of the lesions in Crohn’s disease, the ileum may be affected and ileal absorptive function may be compromised.


 The Schilling test, in two parts (i.e., without and with the addition of oral IF), can help localize the mechanism of vitamin B12 malabsorption.


 Vitamin B12 deficiency results in a macrocytic anemia, often accompanied by hypersegmented neutrophils and neurologic symptoms (e.g., decreased proprioception).2


6. A. Type 1 diabetes.
Rationale: Disorders of the endocrine pancreas do not generally affect the exocrine function of the gland.
B. Crohn’s disease.
Rationale: Crohn’s disease may alter gastrointestinal absorption of d-xylose, thereby causing decreased urinary d-xylose. This is the reason the d-xylose test is performed: to check for defects in enteric absorption.
C. Cystic fibrosis.
Rationale: In cystic fibrosis, the classic finding is insufficiency of the exocrine pancreas. d-Xylose does not require any pancreatic enzymes for absorption. This is how the test differentiates between enteric and pancreatic causes of malabsorption.
D. Chronic renal insufficiency.
Rationale: The d-xylose test depends on the ability of absorbed d-xylose to pass, unaltered, from the blood into the urine. In patients with chronic renal insufficiency, urinary d-xylose is affected by the renal dysfunction, thereby making it difficult to interpret the test results appropriately.
E. Hyperthyroidism.
Rationale: This disorder has no known effect on d-xylose test results.


Major points of discussion


 In the d-xylose test, the patient is given an oral dose of d-xylose after an overnight fast, and then urinary excretion of this molecule is monitored.


 The purpose of this test is to differentiate enteric causes of malabsorption (e.g., ileal dysfunction) from pancreatic causes (e.g., tumor, fibrosis, surgery).


 In patients with malabsorption caused by pancreatic insufficiency (or in a healthy patient), the urinary level of d-xylose should be elevated after the bolus because this sugar does not require pancreatic enzymes for absorption.


 In patients with malabsorption caused by enteric pathology, very little to no d-xylose will be absorbed; therefore, it will not be excreted in the urine.


 Any renal impairment will affect clearance of d-xylose and, therefore, interferes with proper interpretation of the test results.3


7. A. Fecal elastase measurements should be performed only on adult patient samples because pediatric reference ranges do not exist.
Rationale: Reductions in fecal elastase indicate pancreatic insufficiency in children older than 2 weeks.
B. Fecal elastase measurements should not be performed on patients who are receiving pancreatic replacement therapy.
Rationale: Pancreatic enzyme replacement therapy does not interfere with the measurement of fecal elastase.
C. Fecal elastase measurements should be performed only on well-formed stools.
Rationale: Fecal elastase must be performed on well-formed stools or the results have little interpretive value.
D. Fecal elastase measurement is not indicated for the diagnosis of pancreatic insufficiency.
Rationale: Fecal elastase is an extremely good marker for the diagnosis of severe pancreatic insufficiency. It is less sensitive for the detection of milder forms of pancreatic insufficiency.
E. Fecal trypsin, not fecal elastase, measurement should be performed.
Rationale: Fecal elastase is the best marker of pancreatic insufficiency; it is better than measuring other pancreatic enzymes.


Major points of discussion


 Fecal elastase is stable during transit through the bowel.


 Fecal elastase is concentrated in well-formed stools by fourfold to fivefold.


 The test is adequate for the diagnosis of severe pancreatic insufficiency but loses sensitivity in the setting of milder deficiencies.


 Fecal elastase is a better biomarker for pancreatic insufficiency than other pancreatic enzymes in stool.


 This biomarker is not affected by exogenous oral enzyme replacement.39


8. A. Urease breath testing for H. pylori is notoriously inaccurate and should not be used in clinical testing protocols.
Rationale: When performed correctly, urease breath testing is a specific and noninvasive method to confirm positive serologic testing for H. pylori. This is its appropriate clinical use.
B. The endoscopic biopsy was contaminated with H. pylori.
Rationale: Although this thought is not unreasonable, it is highly unlikely. On biopsy, H. pylori organisms are seen deep within the gastric gland pits and are unlikely to be a contaminant.
C. The patient’s use of bismuth subsalicylate caused a false-negative test result.
Rationale: Bismuth-containing compounds are notorious for lowering the burden of H. pylori in patients to below the technical limits of the urease breath test assay. However, these compounds do not eradicate H, pylori and the patient’s bacterial load will rebound, which leads to persistent infection.
D. The patient was infected with a non–urease-producing strain of H. pylori.
Rationale: Although this is possible, these strains are quite rare.
E. Urease breath testing is not useful in patients of Indian descent.
Rationale: There is no evidence of a differential utility of urease breath testing by patient ethnicity.


Major points of discussion


 H. pylori infection is the major cause of peptic ulcer disease worldwide.


 Other causes of peptic ulcer disease include chronic nonsteroidal antiinflammatory drug (NSAID) use and hypersecretory states.


 Chronic persistent H. pylori infection predisposes patients to increased risk for gastric carcinoma and gastric mucosal associated lymphoid tissue (MALT) lymphomas.


 False-negative results of the urease breath test include use of bismuth-containing antacids, PPIs, and antibiotics and testing the patient too soon after the completion of a treatment course. All of these settings may decrease the infecting load of H. pylori below the limits of detection but do not eradicate the infection.


 Urease-negative strains of H. pylori exist but are quite rare.29


9. A. A large spike in glucose concentrations because the lactose is absorbed and broken down into its constituent monosaccharides in the serum by serum lactase.
Rationale: Lactose cannot be absorbed as a disaccharide and must be hydrolyzed by lactase in the intestinal brush border.
B. A small increase in serum glucose caused by residual lactase activity, but most lactose will not by hydrolyzed by lactase in the intestinal brush border.
Rationale: Most lactose-intolerant individuals have some residual enzymatic activity resulting in a small spike in serum glucose levels. This increase must be less than 20 mg/dL to be diagnostic for lactose intolerance.
C. A small increase in serum glucose because of cross-reactivity between lactose and glucose.
D. A large decrease in serum glucose concentrations because most assays are inhibited by lactose.
E. A small decrease in serum glucose concentrations because most assays are inhibited by lactose.
Rationale: No lactose is absorbed.


Major points of discussion


 Lactose intolerance is caused by a decrease in the amount or function of the enzyme lactase in the brush border on the apical surface of small intestinal epithelial cells.


 Lactase acts on the disaccharide lactose to cleave it into its constituent monosaccharides: glucose and galactose. These sugars are then absorbed.


 The gold standard for the diagnosis of lactase deficiency is enzymatic testing of an endoscopically obtained biopsy of the small intestinal mucosa.


 A large percentage of otherwise normal adults are lactose intolerant. They are typically diagnosed by symptoms alone and managed by avoiding dairy products.


 Lactose intolerance is more common in patients of African or Asian descent.13


10. A. A low percentage of C14 on the first round of testing and a relative increase in C14 percentage when C14 is coadministered with pancreatic enzymes.
Rationale: This is the typical pattern seen in pancreatic insufficiency. The first round of testing involves having the patient fast overnight and administering an oral bolus of lipids with a C14-labeled glycerol backbone. In steatorrhea of any etiology, very little C14 will be absorbed and metabolized to C14-CO2. The percentage of total CO2 will be low. However, in cases of pancreatic insufficiency with addition of pancreatic lipases, the glycerol is liberated and absorbed and metabolized to CO2, thereby increasing the relative percentage of C14-labeled CO2 that is expired.
B. A low percentage of C14 on the first round of testing without any change in C14 percentage after administration of pancreatic enzymes.
Rationale: This is the pattern seen in enteric causes of fat malabsorption.
C. A high percentage of C14 on the first round of testing without any change in C14 percentage after administration of pancreatic enzymes.
Rationale: This is the pattern seen in a normal patient with no fat malabsorption.
D. A high percentage of C14 on the first round of testing with a decrease in C14 percentage after administration of pancreatic enzymes.
E. A low percentage of C14 on the first round of testing with a decrease in C14 percentage after administration of pancreatic enzymes.
Rationale: It is difficult to interpret these results. If these results are obtained, the tests should be repeated.


Major points of discussion


 Steatorrhea is the term given to malabsorption of fats.


 Typically, steatorrhea presents with flatulence, bloating, and foul-smelling diarrhea with high fat content that floats on water.


 Generally, 93% of dietary fats are absorbed in a healthy gastrointestinal tract. In steatorrhea, as much as 40% of dietary fat is not absorbed and is passed in the stool.


 The causes of fat malabsorption can be divided into those that are primarily pancreatic in nature (e.g., a lesion obstructing the pancreatic duct or ampulla of Vater) or enteric (e.g., primary small intestinal pathologies).


 A common cause of steatorrhea, particularly in a patient with a camping history, is infection with Giardia lamblia.25


11a. A. A stool osmotic gap of less than 50 mOsm/Kg (low) and pH less than 5.5.
Rationale: A stool osmotic gap of less than 50 is typically seen in secretory diarrheas.
B. A stool osmotic gap of more than 125 mOsm/kg (high) and pH greater than 5.5.
Rationale: In osmotic diarrheas, the nonabsorbable material alters the osmotic gradient, drawing water into the lumen. This decreases the stool water sodium and potassium concentrations and increases the stool osmotic gap (normally, 80 mOsm/kg). A stool pH less than 5.5 (acidic) is classically seen in carbohydrate malabsorptive diarrhea.
C. A stool osmotic gap of 80 mOsm/kg (normal) and pH less than 5.5.
Rationale: A normal osmotic gap with an acidic pH is not typical of an osmotic diarrhea.
D. A stool osmotic gap of more than 125 mOsm/kg (high) and pH of 7.
Rationale: Although this osmotic gap correlates with an osmotic diarrhea, the neutral pH is not classic for a carbohydrate malabsorptive processs, which is generally characterized by an acidic stool caused by increases in various organic acids in the gut owing to bacterial metabolism of the carbohydrates.
E. The stool osmotic gap is not a useful assay to discriminate between secretory and osmotic diarrhea.
Rationale: The large differences in classic osmotic gaps between these two types of diarrhea make osmotic gap testing ideal for this differential diagnosis.


11b. A. Stool water sodium level.
B. Stool water potassium level.
C. Stool pH.
Rationale: Stool pH values cannot be interpreted in the setting of coadministered antibiotic therapy because of changes in gut flora that might dampen the effect of bacterial carbohydrate metabolism on stool pH.
D. Stool osmotic gap.
E. Stool water chloride level.
Rationale for A, B, D, and E: Unless the antibiotic is known to affect electrolyte balance in the intestine, there should be no effect of the coadministered antibiotics on this analyte.


Major points of discussion


 Stool osmotic gap is calculated by taking a value of 290 (a value of stool osmolality that approximates that in the blood) and subtracting from it 2 times the value of the sodium and potassium measured in the stool water. The factor of two accounts for the paired anions for sodium and potassium. A normal stool osmotic gap is 80 mOsm/kg.


 An osmotic gap greater than 125 mOsm/kg suggests an osmotic diarrhea (e.g., malabsorptive processes).


 An osmotic gap less than 50 mOsm/kg is seen in secretory diarrheas.


 In carbohydrate malabsorption, the stool pH is generally less than 5.5. It is acidic because of organic acids (including lactic acid) produced by bacterial metabolism of the poorly absorbed carbohydrate (e.g., lactose).


 Stool osmolality increases if the specimen is stored at room temperature, causing a false decrease in the osmotic gap. Therefore, the sample must be assayed immediately after collection.6


12. A. Gastrin-secreting tumor secondary to Zollinger-Ellison syndrome.
Rationale: Serum gastrin levels in patients with Zollinger-Ellison syndrome are generally greater than 1000 ng/mL. The diagnosis of Zollinger-Ellison syndrome is made not on gastrin levels alone but also after measuring gastric pH and imaging the pancreas.
B. Falsely elevated result caused by prolonged storage at room temperature before testing.
Rationale: Gastrin levels in serum should be run immediately or be immediately frozen at −70°C to prevent proteolytic degradation of the enzyme. In fact, after 48 hours at 4°C, 50% of the gastrin immune reactivity is lost.
C. Atrophic gastritis.
Rationale: Although gastric atrophy does lead to higher gastric pH and an increase in circulating gastrin levels, this is not the most likely answer in this clinical setting.
D. Normally elevated postprandial serum gastrin levels.
Rationale: Peptide fragments produced by digestion of proteins in food induce gastrin secretion. These are two to four times higher after a meal. Representative gastrin levels should be drawn on a fasted sample.
E. Serum gastrin is not the appropriate biomarker for Zollinger-Ellison syndrome.
Rationale: The triad of peptic ulcer disease, elevated serum gastrin levels, and hyperchlorhydria are diagnostic for Zollinger-Ellison syndrome.


Major points of discussion


 One cause of elevated gastrin levels in patients is Zollinger-Ellison syndrome. This entity is defined as a triad of elevated serum gastrin levels (usually > 1000 ng/mL), peptic ulcer disease, and hyperchlorhydria.


 Serum gastrin levels increase with age as a result of progressive gastric atrophy.


 Gastrin secretion is highest at a gastric pH of 5 to 7. Gastrin secretion decreases by a negative feedback mechanism at pH less than 2.


 Increases in serum gastrin levels are common in patients taking PPIs whose gastric pH is generally elevated. However, gastric pH testing would reveal the absence of a hypersecretory state.


 After a meal, gastrin levels increase to two to four times of the upper limit of the reference range.16


13. A. Activation of the renin-aldosterone system.
Rationale: Patients with end-stage cirrhosis can have severe ascites and third spacing caused by low plasma oncotic pressure. This depletes their intravascular volume and activates the renin-aldosterone system to preserve volume. As a result, electrolyte disturbances, including hyponatremia or hypernatremia, are common in this patient population.
B. Overhydration.
Rationale: Overhydration is possible but is not the most common mechanism for electrolyte disturbances in patients with liver failure.
C. Assay interference by elevated bilirubin.
Rationale: Assay interference is possible but is not the most common mechanism for electrolyte disturbances in patients with liver failure.
D. Nutritional imbalance.
Rationale: Nutritional imbalance very rarely causes hyponatremia.
E. Normal variation in serum sodium.
Rationale: Serum sodium is very tightly regulated and is not subject to wide intraindividual variation.


14a. A. Acute or chronic alcohol abuse.
Rationale: This patient has acute pancreatitis. There are no laboratory signs of alcohol abuse, including a normal AST/ALT ratio of about 1. In addition, the lipase level would be expected to be fivefold greater than the amylase level.
B. Interference by statins.
Rationale: This patient has acute pancreatitis. There is no association between statin use and interference with serum amylase activity. Hypertriglyceridemia could cause a false decrease in serum amylase activity, but this patient has normal lipid studies.
C. Collection of the patient’s blood sample in heparin anticoagulant.
Rationale: Serum amylase measurement depends on its enzyme activity, which requires calcium. Heparin tubes are recommended to avoid calcium chelation because amylase enzyme activity requires calcium and its chelation would lower this activity.
D. Collection of the patient’s blood sample in K2EDTA.
Rationale: Serum amylase measurement depends on its enzyme activity, which requires calcium. EDTA and citrate anticoagulants chelate calcium; collection in these tubes could give a false normal or low activity. Therefore, amylase should be checked again using blood collected in a heparinized tube.
E. Storage of the sample at room temperature for 4 hours.
Rationale: Specimens collected for amylase and lipase can be stored for up to 14 days at room temperature.


14b. A. Serum amylase.
Rationale: Amylase can be normal in alcohol-induced acute pancreatitis.
B. Lipase/amylase ratio greater than 3.
Rationale: A lipase/amylase ratio greater than 5 is diagnostic for alcohol-induced acute pancreatitis.
C. Carbohydrate-deficient transferrin.
Rationale: Excessive alcohol consumption leads to elevated carbohydrate-deficient transferrin levels for several days to weeks after binge drinking. It is a useful test for patients who deny alcohol use, but in whom the physician still suspects it.
D. Total and direct bilirubin.
Rationale: Bilirubin studies are nonspecifically elevated depending on the severity of acute pancreatitis. In fact, an elevated direct bilirubin, indicative of obstruction, would support a diagnosis of biliary pancreatitis, rather than alcohol-induced pancreatitis.
E. Trypsinogen-1.
Rationale: Trypsinogen-1 elevation, as well as serum amylase and lipase elevation, suggests biliary pancreatitis. However, some data support the concept that elevated trypsinogen-2 and α1-antitrypsin levels support the diagnosis of alcohol-induced pancreatitis.


Major points of discussion


 Many conditions can cause elevated serum amylase; these most commonly include conditions such as diabetic ketoacidosis (~ 60% of cases), cholecystitis, and peptic ulcer, following episodes of viral hepatitis or ruptured ectopic pregnancy, or after procedures such as gastric resection or renal transplantation.


 Normal serum and urine amylase is present in 20% of cases of acute pancreatitis. These can be false-negative results, such as that noted in Question 14a, but there are common clinical causes. For example, hyperlipidemia is thought to suppress serum amylase activity.


 In acute pancreatitis, serum lipase levels rise slightly earlier (4 to 8 hours) and peak sooner (24 hours) than serum amylase (rises at 2 to 12 hours, peaks at 48 hours).


 Prolonged elevation of serum lipase longer than 14 days suggests a poor prognosis or a pseudocyst.


 Elevated serum amylase and lipase can occur following abdominal trauma. Thus, a diagnosis of acute pancreatitis should be made only if it is supported by the total clinical picture.


 Pancreatic amylase (i.e., P-type) has a molecular weight of 54,000, whereas that of salivary amylase (S-type) can be larger. The enzymatic assay for salivary amylase requires the presence of 10 mmol/L of chloride. Interestingly, both serum and urine P-type amylase are elevated in acute pancreatitis, whereas S-type amylase activity is decreased in chronic relapsing pancreatitis and cancer of the head of the pancreas.


 The Ranson criteria are useful for predicting the severity of acute pancreatitis. Thus, on admission, severity is predicted by factors such as age older than 55 years, white blood cell count greater than 16,000 cells/mm3, blood glucose higher than 200 mg/dL, serum AST higher than 250 IU/L, and serum LDH higher than 350 IU/L. As in this patient, severity at 48 hours is predicted by factors such as calcium lower than 8.0 mg/dL, hematocrit decreased by more than 10%, hypoxemia with Po2 less than 60 mm Hg, blood urea nitrogen increased by at least 5 mg/dL after administration of intravenous fluids, base deficit more than 4 mEq/L, and sequestration/third spacing of fluids greater than 6 L.28


15. A. Perform a repeat small intestinal biopsy.
Rationale: Although a biopsy would show mucosal and submucosal changes that indicate effective treatment (i.e., decreased intraepithelial inflammation and reversal of villous flattening), this is an unnecessarily invasive test. Serologic studies of anti-endomysial immunoglobulin A (IgA) antibodies are less invasive, have a diagnostic sensitivity of 89% to 100% in adults, and disappear after diet modification.
B. Measure serum anti-endomysial IgA antibodies.
Rationale: Serum anti-endomysial IgA antibodies disappear after effective treatment of celiac sprue.
C. Measure serum anti-endomysial IgG antibodies.
D. Measure fecal fat and fecal leukocytes.
Rationale: Although celiac sprue can cause steatorrhea and fecal leukocytes may be present, they are not used to monitor dietary modification. Multiple factors can change fecal fat and leukocytes. Therefore, serologic studies for serum anti-endomysial IgA antibodies are more specific.
E. Measure serum anti-gliadin IgG antibodies.
Rationale for C and E: Serum anti-gliadin IgG antibodies are accurate for diagnosing celiac sprue but inaccurate for monitoring treatment response. In general, IgG serologies for celiac sprue are not useful for monitoring dietary modification.


Major points of discussion


 Four serum antibodies can be used to diagnose celiac sprue: anti-gliadin IgA and IgG, anti-endomysial IgA and IgG, serum anti-reticulin (ARA) IgA, and serum transglutaminase (tTG) IgA. Of those, anti-endomysial IgA has the highest sensitivity and specificity.


 Serum anti-gliadin IgG is currently the most accurate alternative serologic test in patients who have IgA deficiency. However, small intestinal biopsy is still the gold standard.


 Anti-endomysial antibody detection is performed by allowing the antibodies to bind connective tissue that surrounds smooth muscle. Current tests use human umbilical cord sections as a substrate.


 Biopsy of small intestine is the gold stand for diagnosing celiac sprue.


 Nonspecific hematologic and biochemical abnormalities secondary to enteropathy and malabsorption can be present in patients with celiac sprue.



16a. A. Focal segmental glomerulosclerosis.
Rationale: This child has signs of renal failure and nephrotic syndrome with edema secondary to protein loss and low albumin. However, focal segmental glomerulosclerosis causes nephrotic syndrome primarily in adults.
B. Malnutrition.
Rationale: Malnutrition is one manifestation of cystic fibrosis, secondary to pancreatic insufficiency. However, it causes an elevated chloride sweat test (i.e., > 60 mmol/L). Thus, if this patient does not have cystic fibrosis but is poorly nourished, a false-positive sweat chloride test result is possible.
C. Minimal change disease.
Rationale: False-negative chloride sweat test results have been reported in cases of hypoproteinemic edema which, in this case, is caused by nephrotic syndrome. Minimal change disease is the most common cause of nephrotic syndrome in the pediatric setting.
D. Glucose-6-phosphate dehydrogenase (G6PD) deficiency.
Rationale: G6PD deficiency causes elevated sweat chloride to levels above 60 mmol/L. The patient does not have signs and symptoms of G6PD deficiency. Whites of Northern European descent rarely have G6PD deficiency; it is most common among those of African, Southeast Asian, and Mediterranean descent.
E. Previous fludrocortisone administration.
Rationale: Fludrocortisone is sometimes used during chloride sweat testing for cystic fibrosis. It decreases sweat chloride in patients without cystic fibrosis but does not affect sweat chloride in patients with cystic fibrosis. The patient does not have signs of adrenal insufficiency (e.g., hypokalemia) that would indicate the use of fludrocortisone. In addition, the patient clinically shows signs of cystic fibrosis, including a history of upper respiratory infections. Therefore, it is unlikely that the child would have been given steroids, which could cause immunosuppression.


16b. A. The sensitivity is 80% even in African Americans.
Rationale: The sensitivity of molecular testing for cystic fibrosis in this population is approximately 50%.
B. The sensitivity is 75% to 90% in non-Ashkenazi white North Americans.
Rationale: The sensitivity of molecular testing for cystic fibrosis in this population is actually 75% to 90%. Although it is not absolutely necessary in this patient, it could be useful because this child has nephrotic syndrome and the sweat chloride test result is falsely low. Furthermore, there seems to be a family history of cystic fibrosis. The mother can refuse the test, and an elevated sweat chloride test result should be followed after the nephrotic syndrome resolves. Thus, the main role of the physician in this case is to accurately educate the patient’s family.
C. The sensitivity is below 50% in non-Ashkenazi white North Americans.
Rationale: With the most current 25-allele–screening panel, the sensitivity of molecular testing for cystic fibrosis is 75% to 90% in non-Ashkenazi white North Americans, 97% in Ashkenazi Jews, 60% in Hispanic Americans, 50% in African Americans, and less than 10% in Asians.
D. The chance that one of the mutations in a 25-allele–screening panel is a deletion of the codon for phenylalanine 508 (Phe-508) in CFTR is 70%.
Rationale: Deletion of the three-nucleotide codon for Phe-508 (ΔF508) accounts for 70% of CFTR mutations in cystic fibrosis. However, currently, there are more than 1300 known additional mutations. Most are rare, and screening for all of them would be very expensive. In the current 25-allele screen, only 7 mutations in addition to the ΔF508 mutation account for at least 1% of cystic fibrosis mutations.
E. Molecular testing for cystic fibrosis is a better screening strategy than the sweat chloride test.
Rationale: Although molecular testing for cystic fibrosis confirms the diagnosis, many mutations are rare, still unknown, or not available for testing. Thus, a negative molecular test does not exclude the diagnosis, particularly if the sweat chloride test result is positive and the patient has signs and symptoms of cystic fibrosis.


Major points of discussion


 A sweat chloride test result of greater than 60 mmol/L on two separate occasions is diagnostic of cystic fibrosis.


 The actual test uses pilocarpine injected into the skin by iontophoresis, stimulating local sweat gland secretion. Sweat is collected by filter paper or gauze (≤ 100 μL), and 20 μL is analyzed using a chloride ion–selective electrode analyzer.


 Preanalytical variables are the most common causes of sweat chloride test errors.


 Sweat chloride tests in adults should be evaluated with caution. Random levels in men can vary up to as high as 70 mmol/L. Cyclic fluctuations occur in premenopausal women, peaking 5 to 10 days before menses.


 Chronic lung disease and malabsorption are the primary clinical problems seen in patients with cystic fibrosis.


17. A. Fecal elastase-1 of less than 100 μg pancreatic elastase/gram of stool and an abnormal ERCP.
B. Fecal elastase-1 of more than 200 μg pancreatic elastase/gram of stool and a normal ERCP.
Rationale: A fecal elastase-1 of greater than 200 μg pancreatic elastase/gram of stool is normal. In conjunction with a normal ERCP, this would argue against a diagnosis of chronic pancreatitis. Fecal elastase-1 is most useful in severe chronic pancreatitis, the most likely diagnosis in this case. ERCP is the radiologic test of choice for chronic pancreatitis.
C. Fecal elastase-1 between 100 and 200 μg pancreatic elastase/gram of stool and normal ERCP.
Rationale: A fecal elastase-1 between 100 and 200 μg pancreatic elastase/gram of stool is consistent with mild to moderate pancreatitis. However, a normal ERCP and nonspecific CT scan would not be diagnostic of chronic pancreatitis. Fecal elastase-1 is most useful in severe chronic pancreatitis, the most likely diagnosis in this case. ERCP is the radiologic test of choice for chronic pancreatitis.
D. Fecal elastase-1 of less than 100 μg pancreatic elastase/gram of stool and normal ERCP.
Rationale for A and D: A fecal elastase-1 of less than 100 μg pancreatic elastase/gram of stool is abnormally low. However, an abnormal radiologic result is required before diagnosing chronic pancreatitis. ERCP is the best radiologic test for chronic pancreatitis. Fecal elastase 1 is most useful in severe chronic pancreatitis, the most likely diagnosis in this case.
E. Fecal elastase-1 of greater than 200 μg pancreatic elastase/gram of stool and an abnormal ERCP.
Rationale: A fecal elastase-1 of greater than 200 μg pancreatic elastase/gram of stool is normal. A low fecal elastase and abnormal ERCP would better support a diagnosis of chronic pancreatitis when combined with ERCP. Chronic pancreatitis is the most likely diagnosis in this case.


Major points of discussion


 A fecal elastase-1 cutoff of less than 200 μg pancreatic elastase/gram of stool has a positive predictive value of only 50%. Thus, radiology and clinical signs and symptoms are also necessary for the diagnosis of chronic pancreatitis.


 The sensitivity of fecal elastase-1 testing in chronic pancreatitis depends on disease severity: It is 100% in severe cases, 77% to 100% in moderate cases, and 0% to 63% in mild cases.


 Compared with chymotrypsin, another assay for pancreatic exocrine function, exogenous enzymes do not interfere with fecal elastase-1 assays. Exogenous enzymes are enzymes in the intestinal tract that come from ingestion of food and enzyme replacement therapy.


 Nonpancreatic gastrointestinal diseases—short gut syndrome, Crohn’s disease, and gluten-sensitive enteropathy—cause false-positive results.


 Current literature indicates that fecal elastase-1 assay is unreliable in patients who have type 1 diabetes mellitus.5,11


18a. A. Gastrin levels will remain unchanged, but the sample should be analyzed immediately.
B. Gastrin levels remain at 95% of their physiologic levels, and the sample is stable for another 5 days.
Rationale: Gastrin levels decrease by about 50% after storage at 4°C for 48 hours. Serum specimens for quantifying gastrin levels (which is performed using an immunoassay) should be frozen immediately at −70°C because activity and stability are lost after storage at 4°C.
C. Gastrin levels will be decreased by about 50%, and the test results will be inaccurate.
D. Gastrin levels will be decreased by about 95%, and the test results will be inaccurate.
Rationale: This is a serum test, and specimens should be collected in a nonadditive glass tube or a serum separator tube. Specimens for quantifying gastrin levels, which is performed using an immunoassay, should be frozen immediately at −70°C and analyzed immediately after thawing. Stability is lost after storage at 4°C, and gastrin levels decrease by about 50% after 48 hours.
E. Gastrin levels will be decreased, but the sample should be analyzed immediately for accurate results.
Rationale for A and E: Serum specimens for quantifying gastrin levels (which is performed using an immunoassay) should be frozen immediately at −70°C because activity and stability are lost after storage at 4°C. Gastrin levels decrease by about 50% after storage at 4°C for 48 hours.


18b. A. No, all gastrin isoform levels are unaffected by meals.
B. No, G17 and G34 gastrin isoform levels, but not the G14 isoform level, are unaffected by meals.
C. Yes, the patient must fast for at least 24 hours before specimen collection.
D. Yes, the patient must fast for 8 hours before specimen collection.
Rationale: The G17 and G34 gastrin isoform levels increase after meals (G17 quadruples and G34 doubles). Therefore, a patient must fast for at least 8 hours before specimen collection.
E. No, high-molecular-weight gastrin isoform levels are the most stable and are unaffected by meals.
Rationale: The opposite is true: High-molecular-weight and incompletely processed isoforms of gastrin, when pathologically increased, cannot be detected by conventional assays. In addition, big gastrin, known as G34, doubles after a meal.


Major points of discussion


 Gastrin has three isoforms in blood and tissues: G34 (big gastrin), G17 (little gastrin), and G14 (mini gastrin).


 All gastrin isoforms are derived from preprogastrin, which is cleaved by trypsin.


 The sensitivity of intraoperative gastrin assays is as high as 88%.


 The optimal pH range for gastrin activity is 5 to 7.


 Gastrin levels higher than 1000 ng/L are diagnostic for a gastrinoma. Zollinger-Ellison syndrome usually causes elevations up to 2000 ng/L.


 False-positive gastrin level results can occur secondary to H2-antagonists and PPIs.


 Gastrin is measured by an enzyme-linked immunosorbent assay (ELISA).


19. A. Failure to rehydrate guaiac-smear slides.
Rationale: Rehydration increases the sensitivity (and lowers the specificity), leading to more false-positive guaiac-smear test results. The slides can be analyzed within 7 days of collection without rehydration and provide accurate results. Furthermore, in this patient, NSAID use and caffeine intake have probably caused gastroesophageal reflux disease, possibly leading to a low-grade bleeding ulcer. Some reports show an association between NSAID use and positive guaiac-smear test when no colon malignancy is present (i.e., false-positive results).
B. Ingestion of chicken.
Rationale: There is no association with white meats and false-positive results on guaiac-smear tests; this is seen only with red meats and some fruits and vegetables.
C. Ingestion of large amounts of vitamin C.
Rationale: Ingestion of large amounts of vitamin C is associated with false-negative guaiac-smear test results. Furthermore, in this patient, NSAID use and caffeine intake have probably caused gastroesophageal reflux disease, possibly leading to a low-grade bleeding ulcer. Some reports show an association between NSAID use and positive guaiac-smear test when no colon malignancy is present (i.e., false-positive result).
D. Ibuprofen-induced gastroesophageal reflux disease.
Rationale: Several drugs can cause a positive guaiac-smear test result. Common ones include NSAIDs, anticoagulants, aspirin, and colchicines. Medications should always be considered if a false-positive test result is suspected.
E. Lack of red meat ingestion.
Rationale: Ingestion of red meat should be avoided because the guaiac-smear test detects the pseudo-peroxidase activity of heme and is, therefore, nonspecific with regard to hemoglobin from humans and animals.


Major points of discussion


 The sensitivity of fecal occult blood testing is 30% to 50%, but an accurate number is uncertain because patients with false-negative results typically do not undergo colonoscopy for confirmation. Thus, we do not know how many test results are truly falsely negative.


 The lower limit of detection for guaiac-smear tests is a 20-mL/day blood loss. A drop in hematocrit or hemoglobin is unnecessary in the setting of a true positive test result. For example, a 1 g/dL hemoglobin decrease over 1 day is more than enough blood loss (i.e., ~ 250 to 300 mL) to yield a positive guaiac-smear test result if the source of bleeding is the gastrointestinal system.


 Fecal occult blood testing should be performed on three consecutive stools, by using two slides each time. Positivity requires a positive result on only one of the two slides.


 Bleeding gums (i.e., ingestion of blood) can lead to false-positive fecal occult blood test results.


 The U.S. Preventive Services Task Force “recommends screening for colorectal cancer (CRC) by using fecal occult blood testing, sigmoidoscopy, or colonoscopy, in adults, beginning at age 50 years and continuing until age 75 years. The risks and benefits of these screening methods vary.”36


20. A. Amylase clearance/creatinine clearance ratio (Cam/CCr).
Rationale: This ratio is very low in type 1 macroamylasemia (< 1%) because it is caused by formation of amylase–macromolecule complexes, which are too large for excretion into the urine.
B. Trypsinogen 2.
Rationale: Trypsinogen 2 is elevated in, but has limited use for, the diagnosis of acute pancreatitis. It is not useful for the diagnosis of macroamylasemia.
C. Lipase/amylase ratio.
Rationale: This ratio has limited utility for the diagnosis of acute alcoholic pancreatitis when the ratio is 5 or greater because of low sensitivity. A minimally elevated ratio is not diagnostic of macroamylasemia.
D. Urine trypsinogen activation peptide.
Rationale: This peptide is elevated in the urine in the early stage of acute pancreatitis and has a 100% negative predictive value in this setting.
E. Carbohydrate-deficient transferrin.
Rationale: Excessive alcohol consumption leads to elevated levels of carbohydrate-deficient transferrin for several days to weeks after binge drinking. It is a useful test for patients who deny alcohol use if the physician remains suspicious. It does not confirm or exclude macroamylasemia.


Major points of discussion


 Macroamylasemia is a disorder of elevated levels of serum amylase without clinical symptoms.


 There are three types of macroamylasemia, differentiated by high (type 1), moderately high (type 2), and trace (type 3) levels of serum macroamylase. The Cam/CCr ratio is low in all three, but very low in type 1.


 The key differential diagnosis for macroamylasemia includes disorders of elevated serum amylase, including pancreatic hyperamylasemia and salivary hyperamylasemia. In the latter two settings, serum macroamylase would be absent.


 The Cam/CCr ratio is not a reliable marker for macroamylasemia when acute or chronic renal failure is present along with an elevated serum amylase. The identification of serum macroamylase itself would be a more accurate diagnostic test in this setting.


 The assay for serum macroamylase uses ultracentrifugation or chromatography.


21. A. Stool sodium of 60 mOsm/kg, potassium of 80 mOsm/kg, and osmolality of 290 mOsm/kg.
Rationale: The patient has factitious diarrhea, best diagnosed by a good clinical history and the presence of an osmotic gap. Osmotic gap = 290 − {2 × ([Na] + [K])}, where less than 50 mOsm/kg indicates a secretory diarrhea, whereas more than 125 mOsm/kg indicates an osmotic diarrhea. Magnesium laxatives cause an osmotic diarrhea. For these laboratory results, the osmotic gap is 10 mOsm/kg, which would support a secretory diarrhea. Sodium phosphate laxatives cause a secretory diarrhea.
B. Stool sodium of 10 mOsm/kg, potassium of 20 mOsm/kg, and osmolality of 290 mOsm/kg.
Rationale: The patient has factitious diarrhea. For these laboratory results, the osmotic gap is 230 mOsm/kg, which is more than 125 mOsm/kg and supports an osmotic diarrhea as caused by magnesium laxatives.
C. Elevated serum gastrin levels.
Rationale: The patient has factitious diarrhea. Elevated gastrin levels are found in Zollinger-Ellison syndrome, which causes a secretory diarrhea.
D. More than 5 g of stool lipids over 24 hours.
Rationale: The patient has factitious diarrhea. The gold standard for steatorrhea is the 24-hour fecal lipid test. Normal is less than 5 g over 24 hours. Steatorrhea is defined as more than 5 g of lipids over 24 hours. Steatorrhea can result from malabsorption syndrome and does not differentiate between osmotic and secretory diarrhea, especially in this case.
E. Presence of fecal leukocytes and blood.
Rationale: The patient has factitious diarrhea. These laboratory results would be consistent with an inflammatory diarrhea caused, for example, by Shigella species.


Major points of discussion


 Secretory, osmotic, malabsorption, maldigestion, and inflammatory are the main types of diarrhea.


 Not only do active ingredients of certain drugs cause diarrhea, but carrier components do so as well. For example, sorbitol, which acts as a carrier, is one such ingredient.


 Chronic diarrhea is defined as more than three loose stools per day for more than 4 weeks. Acute diarrhea lasts less than 2 weeks and rarely requires diagnostic testing.


 Bloody or exudative diarrhea suggests an inflammatory cause (e.g., infectious, inflammatory bowel disease, ischemic colitis, radiation-induced colitis).


 Stool osmotic gap can be calculated, if given the stool sodium and potassium, using the following equation: Osmotic gap 290 − {2 × ([Na] + [K])}, where less than 50 mOsm/kg indicates a secretory diarrhea and more than 125 mOsm/kg indicates an osmotic diarrhea.


22. A. Continue the PPI treatment and repeat the breath test after completing antibiotic therapy.
B. Repeat the hydrogen breath test.
C. Perform endoscopy and biopsy of the esophagus and stomach.
D. Perform a urease test on a stomach and esophageal biopsy.
E. Culture the stomach or esophageal biopsy tissue at 25°C.
Rationale: The patient appears to have peptic ulcer disease, which is most commonly caused by H. pylori. Histology is the gold standard for diagnosis. In addition, H. pylori grows at 37°C. PPIs and antibiotics can lower the bacterial load and yield a false-negative hydrogen breath test result. However, the patient did not complete the full course of antibiotics, and therefore, she is not completely treated. Histology and serologic testing would be better approaches because this patient has already started therapy for H. pylori before confirmatory diagnosis. Accurate diagnosis is necessary for proper treatment. Persistent epigastric pain, even with therapy, could indicate a neoplasm, which requires biopsy.


Major points of discussion


 H. pylori grows at 37°C. In addition, it is oxidase, catalase, and urease positive. On susceptibility testing, it is resistant to nalidixic acid and sensitive to cephalothin.


 Warthin-Starry and Giemsa stains of tissue biopsies are alternatives to immunohistochemistry for the identification of H. pylori.


 Always consider NSAID use as a cause of peptic ulcer disease.


 H. pylori should be excluded before initiating PPI and antibiotic therapy because treatment can obscure the diagnosis, causing false-negative results if subsequent diagnostic tests are conducted. In practice, PPI therapy is often initiated before confirmation.


 A low bacterial load in general, even without treatment, can yield false-negative test results.


23. A. Positive serum P-ANCA and negative serum ASCA.
Rationale: This would help confirm ulcerative colitis because 70% of patients who have ulcerative colitis have a positive serum P-ANCA compared with 15% of those who have Crohn’s disease.
B. Negative serum P-ANCA and positive serum ASCA.
Rationale: This would actually exclude ulcerative colitis because only 20% of patients who have ulcerative colitis have a positive serum ASCA compared with 65% of those who have Crohn’s disease.
C. Negative serum P-ANCA and negative serum ASCA.
Rationale: Less than 5% of patients who have irritable bowelsyndrome have negative P-ANCA and ASCA serologies.
D. Colon and small intestinal biopsy showing discontinuous presence of granulomas, crypt abscesses, and transmural inflammation.
Rationale: Both ulcerative colitis and Crohn’s disease can have granulomas and crypt abscesses. Transmural inflammation and discontinuous lesions (i.e., skip lesions) are consistent with Crohn’s disease.
E. Low levels of serum vitamin B12.
Rationale: Low levels of serum vitamin B12 would suggest damage to the ileum. Crohn’s disease affects the mucosa anywhere from the mouth to the anus, whereas ulcerative colitis affects colon and rectal mucosa. Thus, low levels of vitamin B12 would be more consistent with Crohn’s disease.


Major points of discussion


 The sensitivity of serum P-ANCA and ASCA for ulcerative colitis and Crohn’s disease are low.


 Histologic diagnosis is the gold standard for diagnosing inflammatory bowel disease.


 The presence of fecal leukocytes is consistent with an inflammatory etiology of diarrhea.


 Always note a patient’s clinical signs and symptoms when the diagnostic tests are inconclusive. These will usually support one diagnosis rather than the other.


 Anemia and elevated inflammatory markers (e.g., C-reactive protein and erythrocyte sedimentation rate) are nonspecific inflammatory markers that can be present in both ulcerative colitis and Crohn’s disease.


24. A. Request microbial culture of the small intestinal tissue.
B. Discontinue abacavir therapy for HIV.
C. Order a polymerase chain reaction (PCR) test of small intestinal tissue.
D. Initiate treatment with metronidazole.
E. Measure serum anti-endomysial immunoglobulin A antibodies.
Rationale: This patient has Whipple disease caused by Tropheryma whipplei, an organism that is not readily cultured. Serum anti-endomysial antibody serologic tests are used to confirm celiac sprue, which also causes a malabsorption diarrhea. PCR of the infected tissue, which is the small intestine in this case, confirms the diagnosis of Whipple disease. Treatment includes penicillin, ampicillin, tetracycline (uncommon now), cotrimoxazole, and more recently, doxycycline and hydroxychloroquine. Sulfonamides can be added if neurologic symptoms are present, indicating infection of the central nervous system. Metronidazole, among other uses, is used to treat diarrhea secondary to Clostridium difficile.


Major points of discussion


 Whipple disease is most common among whites, particularly those from rural areas.


 A duodenal biopsy showing foamy macrophages in the lamina propria that are PAS positive, diastase resistant is pathognomonic for Whipple disease.


 PAS-positive macrophages may also be seen in patients with AIDS who are infected with Mycobacterium avium complex. Therefore, PCR testing of the infected tissue is a better confirmatory test.


 T. whipplei can infect the central nervous system, causing neurologic symptoms. Central nervous system infection requires longer-term antibiotic therapy.


 Fifty percent of patients with Whipple disease have lymphadenopathy and hyperpigmentation. Arthropathy, malabsorption, and diarrhea are also common symptoms.


25. A. The patient most likely does not have alcoholic liver disease if the ALT and AST levels are normal.
Rationale: Serum GGT is often elevated in both alcoholic liver disease and alcoholism without liver disease. GGT levels decline with the cessation of alcohol use. ALT and AST levels may be normal in end-stage liver disease with or without alcohol use as the cause.
B. The patient’s 5′-nucleotidase activity would add no additional diagnostic information.
Rationale: 5′-Nucleotidase is very rarely elevated in bone disease and not associated with alcohol use and therefore may help determine whether the elevated ALP is caused in part by biliary obstruction or bone disease alone.
C. The patient’s ALP would show at least partial heat lability on heat fractionation.
Rationale: The patient would be expected to have an elevated ALP isoenzyme derived from bone, which is the most heat-labile ALP, compared with those from placenta, intestine, and liver.
D. The patient has an elevated Regan isoenzyme.
Rationale: Regan isoenzyme is placental ALP, which is sometimes elevated in malignancy.
E. The patient is expected to have an increased indirect bilirubin.
Rationale: The presence of gallstones puts the patient at risk for extrahepatic biliary obstruction, which would result in increased conjugated (direct) bilirubin rather than unconjugated (indirect) bilirubin. Alcoholic liver disease and cirrhosis can also result in increased conjugated bilirubin.


Major points of discussion


 ALP is a hydrolase enzyme produced in liver, bone, kidney, intestine, and placenta. It is most commonly elevated in liver or bone disease. In the liver, the enzyme is found on the canalicular surface of hepatocytes, and therefore elevated ALP indicates biliary obstruction, cholestasis, or space-occupying lesions. Bone ALP is produced by osteoblasts and is elevated in Paget’s disease, metastatic tumors, and metabolic bone disease, among others.


 ALP isoenzymes can be separated by electrophoresis to distinguish the tissue of origin. In addition, heat fractionation can be used to distinguish the tissue source, with placental ALP being the most heat resistant and bone ALP being the most heat labile. Liver-derived ALP is moderately stable. Remember, “bone burns.”


 Placental ALP is known as Regan isoenzyme and is known to be produced by a number of malignancies, including germ cell and urinary tract tumors. Regan isoenzyme is heat stable.


 Both GGT and 5′-nucleotidase help distinguish between an elevated ALP caused by liver versus bone disease and generally may be used interchangeably. However, GGT is also associated with chronic alcohol use and alcoholic liver disease. Therefore, in this case, given that both the elevated ALP and GGT can be explained by other underlying conditions (Paget’s disease and alcoholism, respectively), further laboratory evaluation for hepatobiliary disease is warranted, including AST, ALT, bilirubin, albumin, and prothrombin time.


 The presence of gallstones puts the patient at risk for extrahepatic biliary obstruction, which would result in increased conjugated (direct) bilirubin rather than unconjugated (indirect) bilirubin. Alcoholic liver disease and cirrhosis can also result in increased conjugated bilirubin. Obstruction caused by gallstones would result in elevated ALP, 5′-nucleotidase, and GGT with or without mild elevation in ALT and AST.


 Patients with cirrhosis of any cause often have normal or decreased levels of AST and ALT because ongoing hepatocyte injury is absent. However, the abnormal liver architecture of cirrhosis can result in regional intrahepatic obstruction of bile flow, resulting in elevated ALP, GGT, and 5′-nucleotidase.


26. A. Activation of the renin-aldosterone system.
Rationale: Patients with end-stage cirrhosis, as described in this patient, generally have severe ascites and third spacing caused by low plasma oncotic pressure. This depletes their intravascular volume and activates the renin-aldosterone system to preserve volume. As a result, electrolyte disturbances, including hyponatremia or hypernatremia, are common in this patient population.
B. Overhydration.
Rationale: Overhydration is possible, but it is not the most common mechanism for electrolyte disturbances in patients with liver failure.
C. Assay interference by elevated bilirubin.
Rationale: Assay interferences are possible, but they are not the most common mechanism for electrolyte disturbances in liver failure patients.
D. Nutritional imbalance.
Rationale: Nutritional imbalance very rarely causes hyponatremia.
E. Normal variation in serum sodium.
Rationale: Serum sodium is very tightly regulated and is not subject to wide intraindividual variation.


Major points of discussion


 Patients with end-stage cirrhosis have severe ascites and third spacing caused by low plasma oncotic pressure. This depletes their intravascular volume, activating the renin-aldosterone system to preserve volume. As a result, electrolyte disturbances, including hyponatremia or hypernatremia, are common in this patient population.


 Overhydration is possible, but it is not the most common mechanism for electrolyte disturbances in patients with liver failure.


 Interference in the sodium assay by hyperbilirubinemia is possible, but it is not the most common mechanism for electrolyte disturbances in patients with liver failure.


 Nutritional imbalance very rarely causes hyponatremia.


 Serum sodium is very tightly regulated and is not subject to wide intraindividual variation.



27. A. Autoimmune hepatitis.
Rationale: Autoimmune hepatitis is inflammation of the liver of unknown cause.
B. Bile duct obstruction.
Rationale: Bile duct obstruction can be caused by primary sclerosing cholangitis, mass, or gallstone.
C. Cirrhosis.
Rationale: Cirrhosis is a common endpoint for many liver diseases. It does not cause Dubin-Johnson syndrome.
D. Defective organic anion transporter.
Rationale: A mutation in the canalicular multidrug resistance protein 2 (MRP2) leads to defective anion transport and defective biliary excretion.
E. Acute viral hepatitis.
Rationale: Hepatitis A, B, and C infections lead to hepatocyte injury.


Major points of discussion


 Dubin-Johnson syndrome is a rare autosomal recessive disorder, most commonly seen in the Sephardic Jewish population, associated with persistently elevated conjugated (direct) bilirubin. Patients with Dubin-Johnson syndrome typically have subtle clinical signs and symptoms, most commonly icterus, but also intermittent jaundice and/or constitutional complaints. These clinical features may be apparent only with other concurrent illnesses.


 Dubin-Johnson syndrome is caused by mutations of the gene encoding a multispecific organic anion transporter protein (MRP2). This protein transports organic anions such as conjugated bilirubin into the bile canaliculus from the hepatocytes.


 Although patients with Dubin-Johnson syndrome have elevated conjugated bilirubin, they do not have elevated transaminases, ALP, or GGT. Elevations in those enzymes would suggest intrahepatic cholestatic disease (e.g., medication-related toxicity, infiltrative disease, cholangitis) or extrahepatic obstruction.


 Unconjugated hyperbilirubinemia (elevated indirect bilirubin) may be prehepatic or hepatic. Prehepatic etiologies include increased bilirubin production such as hemolysis or dyserythropoiesis. Hepatic causes include neonatal jaundice, as well as heritable disorders such as Gilbert and Crigler-Najjar syndromes. Of all the heritable bilirubin metabolic disorders, Crigler-Najjar syndrome type I is the most serious and is often fatal, presenting as severe jaundice and kernicterus in neonates.


 Rotor syndrome is similar to Dubin-Johnson syndrome in that it is also a rare genetic disease with a benign course and presents as elevated conjugated bilirubin and normal LFTs. However, the underlying mechanism is defective storage rather than defective transport of organic anions (conjugated bilirubin). In addition, the liver appears unremarkable compared with the dense pigmentation seen in Dubin-Johnson syndrome. Different patterns of urine coproporphyrin excretion can also help discriminate between these two entities.


28a. A. Cirrhosis.
Rationale: Although patients with this clinical scenario in a chronic setting may develop cirrhosis, you would expect normal to low AST/ALT ratio and abnormal markers of synthetic function resulting from replacement of liver parenchyma with fibrosis.
B. Hepatocellular carcinoma.
Rationale: Hepatocellular carcinoma and other space-occupying lesions would likely show a marked increase in ALP and biliary obstruction.
C. Passive congestion.
Rationale: Passive congestion produces mild elevations in hepatocellular markers (AST/ALT ratio) and biliary markers (total bilirubin/ALP). However, synthetic function remains unaltered.
D. Fulminant hepatic failure.
Rationale: Finally, in fulminant hepatic failure, AST and ALT would be markedly elevated (100 times the upper limit of normal). Synthetic function would also be compromised.
E. Normal variation in liver function.
Rationale: Mild elevations in LFTs generally indicate pathology.


28b. A. Myoglobin.
B. Troponin T.
C. Ischemia-modified albumin.
Rationale: Although there may be a small degree of cardiac injury leading to increases of the other three biomarkers, the main elevation will be in brain natriuretic peptide.
D. Brain natriuretic peptide.
Rationale: The patient is clearly suffering from right heart failure, and elevations in brain natriuretic peptide as a result of right ventricular overloading would be expected.
E. LDH.
Rationale: LDH may be elevated in both myocardial and hepatic pathologies. However, this laboratory value would not help elucidate the mechanism of the liver dysfunction in this case.


Major points of discussion


 The pattern of hepatocellular and biliary biomarkers in a patient sample helps determine which hepatic compartment is abnormal.


 Elevated brain natriuretic peptide secondary to ventricular overload suggests congestive heart failure, particularly in this clinical vignette.


 In hepatic cirrhosis, the AST/ALT ratio will be low to normal, and synthetic function will be abnormal because of replacement of liver parenchyma with fibrosis.


 In fulminant hepatic failure, AST and ALT are markedly elevated, sometimes 100 times the upper limit of normal. Synthetic function is compromised.


 Passive congestion, as occurs in right heart failure, produces mild elevations in hepatocellular markers (AST/ALT ratio) and biliary markers (total bilirubin/ALP). Synthetic function remains unaltered.


29. A. Increased serum ceruloplasmin, increased serum total copper, decreased 24-hour urine copper.
Rationale: Serum ceruloplasmin would be decreased and 24-hour urine copper increased in Wilson’s disease.
B. Decreased serum ceruloplasmin, increased serum total copper, increased 24-hour urine copper
Rationale: Serum copper is decreased in Wilson’s disease.
C. Decreased serum ceruloplasmin, decreased serum total copper, decreased 24-hour urine copper.
Rationale: The 24-hour urine copper is increased in Wilson’s disease.
D. Decreased serum ceruloplasmin, decreased serum total copper, increased 24-hour urine copper.
E. Increased serum ceruloplasmin, increased serum total copper, increased 24-hour urine copper.
Rationale: In Wilson’s disease, there is a decrease in both serum ceruloplasmin and serum total copper (a measurement that includes bound and unbound copper) and an increase in urinary copper excretion.


Major points of discussion


 Wilson’s disease can present clinically as a progressive neurologic disease, liver disease (typically chronic but occasionally acute), or a psychological disorder. Typically, patients present in childhood or young adulthood.


 Wilson’s disease is an autosomal recessive disease caused by mutation of the ATP7B gene, located on chromosome 13, which encodes transmembrane copper-exporting adenosine triphosphatases localized to the canalicular membrane of hepatocytes. Defective copper exportation results in accumulation of copper within hepatocytes as well as ineffective formation of ceruloplasmin, the major carrier of copper in the blood. Approximately 90% of total serum copper consists of ceruloplasmin. Therefore, both serum ceruloplasmin and total serum copper are decreased in Wilson’s disease. Conversely, the serum non–ceruloplasmin-bound copper concentration is elevated, as reflected in increased 24-hour urine copper, which measures free copper excretion.


 Ceruloplasmin is an acute-phase reactant and therefore may be normal or elevated in patients with Wilson’s disease. Furthermore, ceruloplasmin may be low owing to other causes such as renal disease or non–Wilson’s disease-related end-stage liver disease. Therefore decreased serum ceruloplasmin and serum total copper are not sensitive or specific laboratory tests to confirm the diagnosis of Wilson’s disease. Increased urinary excretion of copper, although a useful screening modality in diagnosing Wilson’s disease, may be increased in other liver diseases such as autoimmune hepatitis.


 Serum ceruloplasmin less than 20 mg/dL with increased hepatic copper of more than 250 μg/g dry weight is diagnostic of Wilson’s disease. Genetic testing for ATP7B mutations has limited diagnostic utility because more than 30 mutations have been identified.


 Administration of d-penicillamine during the 24-hour urine copper excretion test results in increased copper excretion and can be helpful in distinguishing Wilson’s disease from other causes of increased copper excretion.


 Untreated Wilson’s disease results in development of advanced liver fibrosis and cirrhosis, as well as neuropsychiatric complications, and is ultimately fatal. The mainstay of treatment involves copper chelation therapy, such as with penicillamine, and liver transplantation when necessary.30


30. A. A glucose level of 60 mg/dL indicates a poor prognosis.
Rationale: Acute pancreatitis is often associated with defective insulin release and hyperglycemia. A glucose level higher than 200 mg/dL is an adverse prognostic factor in acute pancreatitis.
B. Amylase is released earlier than lipase and therefore is more sensitive for acute pancreatitis.
Rationale: Both amylase and lipase are released 3 to 8 hours after the initiation of the symptoms and persist for several days after recovery, with amylase tending to normalize in 3 to 6 days and lipase often persisting for 1 to 2 weeks. Lipase in most studies was found to be more sensitive than amylase for the diagnosis of acute pancreatitis.
C. Lipase is more specific than amylase for acute pancreatitis.
Rationale: Lipase is more specific for acute pancreatitis, although elevations can occasionally be seen in other pancreatic and gastrointestinal disorders and in renal failure. Amylase can be elevated in a wide variety of conditions, including salivary disease, gastrointestinal disorders, hepatitis, cirrhosis, gynecologic diseases, chronic alcoholism, pregnancy, trauma, burns, head injury, and renal failure. Because of its poor specificity and slightly lower sensitivity, amylase is no longer recommended for the diagnosis of pancreatitis, except in situations of high suspicion with normal lipase.
D. Most commercially available amylase assays are immunoassays.
Rationale: Most amylase assays rely on enzymatic hydrolysis of starch or starchlike substrates such as hepta-maltose. Immune inhibition with antibodies against salivary amylase, followed by an amylase enzymatic assay, has been used occasionally to measure pancreatic amylase more specifically, but these assays are not widely available.
E. Elastase assays are commonly used to rule out acute pancreatitis.
Rationale: Pancreatic elastase is elevated in acute pancreatitis, but its diagnostic efficiency is not better than lipase, and therefore it is not commonly used. However, decreased levels of fecal elastase can be of value in diagnosing chronic pancreatic insufficiency because it remains un-degraded in stool.


Major points of discussion


 Acute pancreatitis is a common cause of abdominal pain in patients presenting to the emergency department and represents a diagnostic challenge because of variable clinical presentation and lack of specificity of laboratory tests.


 The most common causes are cholelithiasis and alcohol use, with genetic factors playing an important predisposing role. Variants in the following genes have been strongly implicated in susceptibility to pancreatitis: cationic trypsinogen (PRSS1), anionic trypsinogen (PRSS2), serine protease inhibitor Kazal 1 (SPINK1), CFTR, chymotrypsinogen C (CTRC), and calcium-sensing receptor (CASR). Rare causes (< 1%) include hypertriglyceridemia, hypercalcemia, malignancy, autoimmune disorders, vascular abnormalities, and hereditary enzymatic defects.


 Age greater than 55 years is associated with an adverse prognosis. Laboratory values indicating an adverse prognosis include age greater than 55 years, blood glucose higher than 200 mg/dL, white blood cell count (WBC) more than 16 × 109/L, AST higher than 250 U/L, LDH more than 700 U/L, creatinine more than 2 mg/dL, C-reactive protein more than 150 mg/dL, procalcitonin greater than 1.8 ng/mL, albumin less than 2.5 mg/dL, and calcium less than 8.5 mg/dL.


 Various professional organizations, including the American Gastroenterological Association and American Academy of Family Physicians, have published guidelines for the diagnosis of acute pancreatitis and unanimously recommend the use of lipase instead of amylase for initial diagnosis of pancreatitis. Most guidelines recommend a threshold of greater than two to four times the upper limit of normal (ULN), while stressing that early elevation (less than three times normal) should be interpreted in the context of the clinical evolution.


 A recent study7 examined the diagnostic accuracy of lipase and amylase for acute pancreatitis in 3451 patients presenting to the emergency department with acute abdominal pain. Elevations of lipase greater than threefold ULN showed sensitivity, specificity, and area under the curve of 95%, 99%, and 0.996, respectively, whereas the corresponding values for amylase were 64%, 99%, and 0.992, respectively.


 Amylase assays rely on enzymatic hydrolysis of starch or starchlike substrates such as hepta-maltose. A dry-chemistry method uses the natural substrate starch coupled to a dye, which is released by amylase and diffuses to the reagent layer, where it is measured by reflectance spectrophotometry. Some liquid chemistry methods use a series of enzymes to convert the product of the amylase reaction (usually one or more maltose residues) to a measurable compound (usually reduced nicotinamide adenine dinucleotide [NADH] from a G6PD reaction). Other methods use chromogenic maltose oligomers that release a measurable dye, such as 4-nitrophenyl, on reaction with amylase.


 Lipases hydrolyze triglycerides and diglycerides to monoglycerides or glycerol and free fatty acids and typically are assayed by measuring hydrolysis of diglyceride substrates. In some methods, production is measured through a series of reactions leading to production of H2O2. Other methods use special diglyceride substrates that, on lipase reaction, release a chromogen. In addition to pancreatic lipase, other lipases released in the blood include lipoprotein lipase, intestinal lipase, and hepatic lipase. The latter two prefer short-chain fatty acid substrates, whereas pancreatic lipase and lipoprotein lipase can hydrolyze long-chain fatty acids. Lipoprotein lipase interference is eliminated by incorporation of bile acids and a pancreatic lipase cofactor, colipase.7,19,27


31. A. AST.
Rationale: AST is a marker for hepatocyte injury and can be normal in conditions causing canalicular injury.
B. ALT.
Rationale: ALT is a marker for hepatocyte injury and can be normal in conditions causing canalicular injury.
C. Anti–hepatitis B core antibody.
Rationale: This test assesses exposure to HBV.
D. GGT.
Rationale: Concurrent elevations in GGT suggest the elevated ALP is of biliary tract origin.
E. Ammonia.
Rationale: Ammonia levels will be elevated in patients with liver failure, but this test is not used to discriminate origins of ALP.


Major points of discussion


 ALP is present in many tissues, including bone and liver.


 In the liver, ALP is located on the canalicular surface of the hepatocyte, and elevations are seen in patients with cholestatic disease.


 GGT is an enzyme involved in amino acid transport across cell membranes. Elevations occur in cholestatic disease.


 If GGT and ALP are both elevated, the source of the ALP is likely to be the biliary tract.


 Posthepatic biliary obstruction will lead to elevations in ALP, GGT, and 5′-nucleotidase. Causes of posthepatic biliary obstruction include cholelithiasis, primary biliary cirrhosis, and primary sclerosing cholangitis.


32. A. ALT is more abundant in the liver than AST.
Rationale: In general, AST is more sensitive than ALT for liver injury because it is about two to three times more abundant in liver than ALT.
B. ALT is more specific than AST for liver disease.
Rationale: ALT is expressed predominantly in liver, although it is present in smaller amounts in other tissues such as kidney, skeletal muscles, heart, adrenal gland, adipose tissue, neurons, endocrine pancreas, and prostate. AST is widely expressed and therefore less specific for liver disease. Despite its presence in other tissues, isolated ALT elevations are rarely seen in the absence of liver disease.
C. The AST/ALT ratio is usually lower than 2 in severe alcoholic liver disease.
Rationale: Typically, patients with alcoholic hepatitis or severe alcohol-induced liver disease such as cirrhosis have an AST/ALT ratio (De Ritis ratio) greater than 2. Possible mechanisms for the elevation of this ratio include decreased hepatic production of ALT, mitochondrial oxidative damage leading to release of mitochondrial AST, and extrahepatic damage with AST release.
D. The AST/ALT ratio is usually lower than 2 during the acute phase of viral hepatitis.
Rationale: In acute viral or toxic hepatitis, aminotransferase levels are typically quite elevated (> 300 U/L), and the AST/ALT ratio tends to be less than 1.
E. AST half-life in plasma is significantly longer than that of ALT.
Rationale: In general, the half-life of ALT is much higher (about 2 days) than that of AST (< 24 hours), and therefore ALT may predominate after recovery of liver injury.


Major points of discussion


 Two different genes code the AST enzyme. GOT1, which codes for cytosolic AST, is most abundant in red cells and in cardiac and skeletal muscles. GOT2 codes for a mitochondrial form of AST, which is most abundant in hepatocytes. The cytosolic form predominates in the plasma of normal individuals and patients with mild liver disease, whereas the mitochondrial form is released with advanced or severe liver disease.


 Two different genes code the ALT enzyme. GPT codes for the cytosolic form (ALT1), predominantly expressed in the liver, whereas GPT2 codes for a mitochondrial form expressed in skeletal muscle and heart. The GPT gene can be induced in the liver by peroxisome proliferator–activated receptor (PPAR) α agonists, such as fenofibrate, and by conditions associated with increased neoglucogenesis, such as fasting, insulin resistance, type 2 diabetes, the metabolic syndrome, and obesity.


 Both AST and ALT are sensitive markers of liver disease. ALT is more specific and AST is more sensitive for mild liver damage.


 Factors affecting the AST/ALT ratio (De Ritis ratio) include (1) extent of extrahepatic disease, affecting AST more than ALT; (2) severity of damage, with more severe disease leading to release of mitochondrial AST; (3) half-life of AST (< 24 hours) versus ALT (about 2 days); (4) liver synthetic dysfunction, affecting ALT more than AST; (5) extent of ALT induction (e.g., drugs, insulin resistance); (5) levels of circulating pyridoxal-5′-phosphate (P5P) in circulation. Because P5P is an essential cofactor for the aminotransferase reaction with assays that do not supplement the reaction with exogenous P5P, this becomes a rate-limiting factor; the ALT enzyme is more susceptible to low levels of P5P seen, for example, with chronic alcoholism, malnutrition, and renal failure.


 The AST/ALT ratio can be helpful to differentiate causes of liver damage. In normal individuals, the De Ritis ratio is about 0.7 to 1.4. In general, the ratio is increased in acute ischemic, toxic, or alcoholic hepatitis, cirrhosis, liver metastasis, primary biliary cirrhosis, Reye syndrome, and Wilson disease. The De Ritis ratio is usually less than 2 in viral hepatitis (often < 1), autoimmune hepatitis, cholangitis, hemochromatosis, infectious mononucleosis, non-alcoholic steatosis, and in the recovery phase of acute hepatitis.8,26,32


33. A. Preoperative elevation of CEA indicates a poor prognosis.
Rationale: Although elevations of CEA greater than 5 ng/mL are associated with the possibility of distant metastasis, particularly in patients with stage III colon cancer, this patient is a heavy smoker, and therefore mild elevations of CEA may be associated with smoking. About 5% of smokers have CEA elevations greater than 5 ng/mL. In these patients, a cutoff of 10 ng/mL may be more indicative of poor prognosis.
B. Increased CEA after the procedure is highly suggestive of treatment failure.
Rationale: Chemotherapy is often associated with a temporary increase in CEA, possibly released from dying cancer cells. CEA elevations should not be interpreted as failure to respond during the first 4 to 6 months of chemotherapy.
C. An early increase of more than 15% in CEA levels during chemotherapy indicates a better prognosis.
Rationale: CEA elevation during chemotherapy may be associated with a better prognosis because it indicates susceptibility to the chemotherapy. For example, a “CEA flare,” defined as at least a 15% increase of 4 ng/mL or greater followed by a decrease of 15% or greater, had an objective response rate of 73% compared with 11% for patients with CEA increasing at least 15% in two consecutive measurements (Strimpakos, 2010).
D. CEA has good accuracy for screening of early-stage (Dukes A) colon cancer.
Rationale: CEA is more often normal in early-stage (Dukes A) colon cancer, with sensitivity ranging from 8% to 52% and specificity ranging from 55% to 100%. Use of CEA for screening and detection of early colon cancer in healthy individuals is not recommended because of poor sensitivity and specificity.
E. Significant CEA elevations (> 10 ng/mL) are specific for colon cancer.
Rationale: CEA elevations are seen in a variety of benign and malignant gastrointestinal diseases, including esophageal and gastric cancer, diverticulitis, gastric ulcer, atrophic gastritis, pancreatitis, inflammatory bowel diseases, hepatobiliary diseases, chronic renal failure, chronic lung disease, cystic fibrosis, pneumonia, sarcoidosis, and other tumors (e.g., lung, breast, ovary, uterus, salivary).


Major points of discussion


 CEA is a glycoprotein abundantly expressed in the fetal gastrointestinal tract, but its expression is shut down in most adult cells. However, it can be re-expressed at high levels in epithelial carcinomas. The CEA protein is anchored to the cell membrane by a glycosyl phosphatidyl inositol tail and is easily cleaved from the membrane by phospholipases expressed in cancer cell, resulting in solubilization of CEA and consequent release into circulation.


 CEA is the only tumor marker recommended by several expert organizations for clinical use in gastrointestinal tumors. Moreover, these guidelines recommend that CEA measurements should be used for staging, therapy monitoring, and recurrence prediction in patients with colorectal cancer.


 CEA should not be used for screening of early-stage colon cancer in healthy individuals because of poor sensitivity and specificity. However, it may be of some value in individuals at high risk for epithelial cancer, with the caveat that more than 90% of the positive CEA results in asymptomatic individuals are a result of benign conditions.


 Pretreatment elevations of CEA are increasingly common with advancing colon cancer stage, ranging from 8% to 52% for Dukes A, 22% to 59% for Dukes B, 38% to 72% for Dukes C, and 69% to 96% for Dukes D. Significant elevations (> 5 ng/mL in nonsmokers or > 10 ng/mL in smokers) should prompt evaluation for the presence of metastatic cancer.


 The major role of CEA testing is evaluation of patients with colon cancer after treatment, with the goals of (1) documenting the effectiveness of treatment and (2) monitoring for recurrence or metastatic disease. It is important to note that CEA should not be the sole test used for these purposes because sensitivity is about 64% to 84%, whereas specificity varies between 73% and 91%. Most important, the vast majority of CEA elevations are false positive in the absence of positive imaging studies and should not be used in isolation to determine aggressive therapy. Conversely, a combination of CEA and imaging should provide more than 90% sensitivity for detection of postsurgical colon cancer recurrences.


 Successful surgical treatment of colon cancer should result in a decrease of CEA 3 to 4 weeks after surgery. In contrast, if chemotherapy with certain agents such as 5-fluorouracil, levamisole, irinotecan, and oxaliplatin is used, temporary “CEA flares” may be observed and should not be equated with treatment failure. Success of chemotherapy should result in decreases in CEA levels after 6 months of therapy.


 Most guidelines recommend CEA measurement every 2 to 3 months in patients with stage II to IV disease for 2 to 3 years, if they are candidates for surgery or systemic treatment in the event of cancer recurrence or metastatic disease. After 2 to 3 years, monitoring every 6 months for another 2 to 5 years is appropriate.31,33,34


34a. A. Unconjugated bilirubin.
Rationale: Unconjugated bilirubin is usually not directly measured but rather is calculated by subtracting the direct from the total bilirubin values.
B. Azobilirubin.
Rationale: Azobilirubin is produced when bilirubin present in the patient’s sample reacts with a diazotized sulfanilic acid. Azobilirubin absorbs light at 540 nm.
C. Nicotinamide adenine dinucleotide phosphate (NADP).
Rationale: NADP will absorb ultraviolet light at a specific wavelength; however, the production of NADP is not used in this reaction methodology.
D. Nicotinamide adenine dinucleotide (NAD).
Rationale: NAD molecules will absorb ultraviolet light; however, the production of NAD is not used in this reaction methodology.
E. Uridine diphosphate (UDP) glucuronyltransferase.
Rationale: UDP glucuronyltransferase is the enzyme that conjugates bilirubin to glucuronic acid. It is not directly measured in this assay.


34b. A. UDP glucuronyltransferase.
Rationale: UDP glucuronyltransferase is the enzyme that conjugates bilirubin to glucuronic acid. It is not directly measured in this assay.
B. Albumin.
Rationale: Albumin levels are not measured with this technique. A bromocresol green assay is commonly used to measure albumin.
C. Unconjugated bilirubin only.
Rationale: Unconjugated bilirubin is detected with this assay; however, conjugated bilirubin present in the sample will also react.
D. Conjugated bilirubin only.
Rationale: Conjugated bilirubin is detected with this assay; however, because of the presence of the accelerators, unconjugated bilirubin present in the sample will also react.
E. Total bilirubin.
Rationale: The addition of accelerants facilitates the measurement of unconjugated and conjugated bilirubin at the same time, yielding a total bilirubin value.


Major points of discussion


 Bilirubin is commonly measured using the diazo method, which results in azobilirubin production. Azobilirubin production is then measured spectrophotometrically because the compound absorbs light at 540 nm.


 Conjugated bilirubin (direct) can be measured using the diazo technique without accelerators. Direct bilirubin reacts more quickly than indirect bilirubin.


 When accelerators such as caffeine are added to the diazo method, both direct and indirect bilirubin will react. This reaction will give a total bilirubin value.


 Indirect bilirubin is calculated by subtracting direct from total bilirubin.


 UDP glucuronyltransferase is the enzyme that conjugates bilirubin to glucuronic acid. Conjugated bilirubin is water soluble and can be filtered by the kidney glomerulus.


35. A. Plasma hemoglobin levels.
Rationale: Plasma hemoglobin is increased by in vitro hemolysis and can be increased in intravascular hemolysis when hemoglobin levels exceed the clearing capacity of the haptoglobin/phagocytic system (about 100 to 200 mg/dL). With in vitro hemolysis, plasma hemoglobin levels become visible at about 20 to 50 mg/dL (corresponding to about 0.1% to 0.5% hemolysis).
B. Plasma potassium levels.
Rationale: Potassium can transiently increase in severe acute intravascular hemolysis, but homeostatic mechanisms including renal excretion will rapidly lead to normal levels.
C. Plasma AST levels.
Rationale: AST is significantly more abundant than ALT in erythrocytes and other blood cells and therefore will increase more than ALT both in vitro and in vivo hemolysis.
D. Plasma LDH levels.
Rationale: LDH is a major intracellular component in blood cells and will increase with both in vivo and in vitro hemolysis. Typically, expect about a 1% increase in LDH for each 1 mg/dL of plasma hemoglobin with in vitro hemolysis.
E. Plasma haptoglobin levels.
Rationale: Hemoglobin released in circulation from intravascular hemolysis will quickly bind to haptoglobin, and the haptoglobin-hemoglobin complexes are rapidly cleared by the phagocytic system. Therefore, a decrease in haptoglobin levels is consistent with in vivo hemolysis. Because there is no significant endocytosis of haptoglobin-hemoglobin complexes in an anticoagulated blood sample in vitro, plasma haptoglobin levels are unchanged by in vitro hemolysis.


Major points of discussion


 Clostridium perfringens sepsis is a cause of acute intravascular hemolysis, in part owing to production of a hemolysin by this anaerobic gram-positive bacillus. Hemolysis can be severe, with visible hemoglobinemia, hemoglobinuria, and jaundice. Blood smears often show pancytopenia, with spherocytosis and ghost cells, and toxic changes in neutrophils. Mortality is very high.


 Distinguishing in vitro from in vivo hemolysis is essential for proper diagnosis of the patient’s condition, as well as for correct interpretation of laboratory values.


 Hemoglobinuria, decreased haptoglobin, increased unconjugated bilirubin and urinary urobilinogen, and reticulocytosis can occur only with in vivo hemolysis and are not seen with artifactual in vitro hemolysis.


 Conversely, false elevations of potassium and, to a lesser extent, phosphate, magnesium, and other intracellular electrolytes, are more often observed with in vitro hemolysis because physiologic homeostatic mechanisms tend to strictly control electrolyte levels in vivo.


 Significant elevations of AST and LDH without ALT elevations are consistent with hemolysis because ALT is not significantly present in red blood cells.


 Whereas a haptoglobin decrease is a sensitive marker for intravascular and, to a lesser extent, extravascular hemolysis, it suffers from the caveat that haptoglobin is an acute-phase reactant and increases in conditions such as stress, infection, acute inflammation, and tissue necrosis. A normal C-reactive protein level can help rule out an acute-phase reaction.


 Other caveats of using haptoglobin for the diagnosis of hemolysis include the fact that it decreases with estrogen and liver disease and increases with corticosteroids and androgens. Situations of phagocytic system blockade, such as viral infections and infiltrative disorders of lipid or glycogen metabolism, may also result in impaired clearing of haptoglobin-hemoglobin complexes.4,18,21


36. A. Hemolytic anemia.
Rationale: Both hemolytic anemia (increased bilirubin production) and conjugation defects (such as UGT1A1 defects and liver cirrhosis) result in mildly increased unconjugated bilirubin, as in this case. However, with increased heme catabolism, the liver is able to increase bilirubin conjugation and excretion, leading to higher levels of urobilinogen production and ultimately positive urine urobilinogen. In contrast, with conjugation defects, as well as with cirrhosis and extrahepatic cholestasis, urobilinogen levels are decreased because conjugated bilirubin excretion into the intestines is impaired.
B. Gilbert syndrome.
C. Crigler-Najjar syndrome.
Rationale: Urobilinogen levels are decreased because conjugated bilirubin excretion into the intestines is impaired.
D. Liver cirrhosis.
Rationale: When cirrhosis is accompanied by jaundice, it is predominantly associated with increased direct bilirubin, resulting from intrahepatic cholestasis, typically with negative urine urobilinogen and possibly positive urine bilirubin.
E. Extrahepatic cholestasis.
Rationale: Extrahepatic cholestasis typically results in increased conjugated (direct) bilirubin in plasma, predominating over unconjugated bilirubin. Urine bilirubin can be elevated, and urine urobilinogen is negative.


Major points of discussion


 Bilirubin derives from the catabolism of hemoglobin, more specifically from its heme moiety. With intravascular or extravascular hemolysis, the phagocytic system converts increased amounts of heme to bilirubin, which then is released into circulation, mostly in a noncovalent complex with albumin (unconjugated or “indirect” bilirubin). This high-molecular-weight complex does not typically appear in urine if glomerular function is intact.


 The liver then conjugates bilirubin to glucuronic acid, a reaction catalyzed by the UDP glucuronyltransferase enzyme coded by the UGT1A1 gene. Congenital defects in this gene lead to forms of Gilbert and Crigler-Najjar syndromes of hyper-unconjugated bilirubinemia.


 Excretion of conjugated bilirubin occurs through the biliary system into the intestines, where 95% is reabsorbed into the portal circulation and re-excreted by the liver. Very little conjugated bilirubin (measured as “direct” bilirubin) is present in the systemic circulation in the absence of intrahepatic or extrahepatic cholestasis.


 About 5% of intestinal bilirubin is further metabolized to urobilinogen by intestinal bacteria. About 80% of the urobilinogen is excreted in the stool, whereas 20% is reabsorbed into the circulation. Both urobilinogen and conjugated bilirubin are soluble in the plasma and can be excreted by the kidneys.


 Unconjugated bilirubin elevations can result from increased production (increased heme catabolism), such as in hemolysis and inefficient erythropoiesis, and from conjugation defects, such as severe liver insufficiency, genetic defects in UGT1A1 (Crigler-Najjar and Gilbert syndromes), and inhibition of UGT1A1 (e.g., anti-HIV protease inhibitors, oral contraceptives, hyperthyroidism).


 Severe hemolysis can be a cause of jaundice when the hepatic capacity to handle increased bilirubin production is exceeded. With preserved liver function, the degree of total bilirubin elevation in hemolysis is mild, rarely exceeding 5 mg/dL. All of the elevation is attributable to unconjugated (indirect) bilirubin.


 Urine urobilinogen is a sensitive marker for hemolysis, often elevated when total bilirubin is normal, because the liver increases bilirubin conjugation and biliary/intestinal excretion.9,10


37. A. ALT of 10,500 U/L (reference range, < 45) and AST of 8500 U/L (reference range, < 35).
Rationale: This pattern is more suggestive of acute viral hepatitis. An AST/ALT ratio greater than 1 is suggestive of progression of cirrhosis.
B. ALP of 650 U/L (reference range, 110 to 390) and GGT of 35 U/L (reference range, 7 to 49).
Rationale: Elevated ALP with normal GGT is more suggestive of bone disease. In alcoholic patients, GGT is usually elevated.
C. Albumin of 3.2 g/dL (reference range, 3.4 to 4.8) and total protein of 10.4 g/dL (reference range, 6.4 to 8.3).
Rationale: Severe cirrhosis is associated with hypoalbuminemia and polyclonal hypergammaglobulinemia, suggested by the elevated levels of total protein and globulin (total protein albumin).
D. Total bilirubin of 4.3 mg/dL (reference range, 0.2 to 1.1) and direct bilirubin of 3.2 mg/dL (reference range, < 0.2).
Rationale: Elevation of bilirubin consisting predominantly of conjugated (direct) bilirubin is more suggestive of cholestasis. Bilirubin elevations associated with advanced cirrhosis typically are composed of unconjugated hyperbilirubinemia, resulting from hepatic insufficiency, and conjugated hyperbilirubinemia, reflecting intrahepatic cholestasis.
E. Prothrombin time of 12.2 seconds (reference range, 12.0 to 14.3) and platelet count of 350 × 109/L (reference range, 150 to 400).
Rationale: Hepatic insufficiency associated with advanced cirrhosis will lead to insufficient production of clotting factors and prolongation of the prothrombin time. Thrombocytopenia is also a common finding in liver cirrhosis.


Major points of discussion


 Liver cirrhosis is characterized by replacement of the liver parenchyma with fibrous tissue and regenerative nodules, leading to loss of normal architecture and impaired liver function.


 Chronic alcoholics are at risk for liver cirrhosis. Various contributory factors include chronic hepatic damage resulting from consistent alcoholic intake (usually > 60 g/day for more than 10 years), malnutrition, genetic factors, oxidative stress, immunologic mechanisms, and hepatic comorbidities.


 True tests of liver function include prothrombin time, which measures clotting factor synthetic capacity; the ratio of unconjugated bilirubin to conjugated bilirubin, which measures the hepatic ability to conjugate bilirubin; and albumin levels, which reflect the balance of liver synthesis, intravascular volume, and excretion. In cirrhosis, hypoalbuminemia contributes to low intravascular oncotic pressure. In addition, there is systemic vasodilation, lower renal perfusion, water retention associated with elevated levels of antidiuretic hormone, and hyponatremia. All of these factors result in expansion of the extravascular volume resulting in ascites and edema.


 ALT and AST are markers of liver damage. In cirrhosis, elevations of aminotransferases are usually mild or absent. Because ALT is more susceptible to liver synthetic deficiency than AST, which is widely expressed in other tissues, the AST/ALT ratio tends to be more than 2 in liver cirrhosis with chronic hepatic insufficiency. Oxidative mitochondrial damage may also contribute to higher levels of AST.


 Polyclonal hypergammaglobulinemia is common in cirrhosis. Contributory factors include impaired clearing of enteric bacterial organisms and antigens by the liver, chronic inflammation, and possibly loss of B-cell inhibitory regulation. Serum protein electrophoresis typically shows a broad increase in the β and γ regions, the so-called β-γ bridge. Often chronic liver disease also shows a decrease in the α1-, α2-, and β-globulins because of hepatic synthetic insufficiency.


 A common finding with severe liver insufficiency is hepatic encephalopathy. Manifestations include disturbed sleep, altered consciousness, defects in higher mental functions and personality, flapping tremors, rigidity, and hyperreflexia. The pathogenesis has been attributed to high levels of ammonia associated with chronic liver disease. Normally the liver converts ammonia to urea by the Krebs urea cycle, or alternatively couples ammonia to glutamic acid to form glutamine, and liver insufficiency together with portosystemic shunting can result in high levels of ammonia. However, not all patients with high ammonia levels have encephalopathy, and conversely, normal levels of ammonia are seen in about 10% of the patients with hepatic encephalopathy, suggesting that other intestinal compounds cleared by the liver may play a role. There is no correlation between the degree of hyperammonemia and the severity of the encephalopathy; therefore, it is of little value to measure ammonia levels in these patients, with the possible exception of diagnostically unclear situations.


 To diagnose progression to cirrhosis, particularly in chronic viral hepatitis, various investigators have attempted to estimate the degree of liver fibrosis with laboratory tests, frequently using combinations such as the AST/platelet ratio index (APRI), fibroindex (derived from platelet count, AST, and γ-globulins), and a variety of markers of collagen synthesis, including procollagen type III N-terminal peptide, triple-helix domain of type IV collagen, prolyl hydroxylase, and hyaluronic acid. A popular index (FibroTest or FibroSURE) uses a combination of α2-macroglobulin, haptoglobin, apolipoprotein A1, GGT, and bilirubin to derive a value predictive of liver fibrosis and necroinflammatory activity.15,20,37


38. A. Total bilirubin.
Rationale: Hyperbilirubinemia can be a feature of primary biliary cirrhosis; however, this test is not specific for this disease.
B. Anti-SMAs.
Rationale: Anti-SMAs are associated with classic autoimmune hepatitis.
C. ALP.
Rationale: High levels of ALP can be seen in patients with primary biliary cirrhosis; however, this analyte is not specific for this disease.
D. Immunoglobulin M (IgM).
Rationale: Elevated levels of IgM can be seen in patients with primary biliary cirrhosis; however, this finding is not specific for this disease.
E. Antimitochondrial antibodies.
Rationale: An antimitochondrial antibody titer greater than 1:160 is highly predictive of primary biliary cirrhosis.


Major points of discussion


 Primary biliary cirrhosis (PBC) is a cholestatic disorder of unknown etiology. The disease is characterized by inflammation and progressive destruction of the bile ducts within the liver.


 The most sensitive diagnostic marker for PBC is the presence of antimitochondrial antibodies.


 Antinuclear autoantibodies are a heterogeneous group of antibodies detected in a variety of autoimmune disorders, including autoimmune hepatitis and PBC.


 Antimitochondrial antibodies are detected using indirect immunofluorescence microscopy.


 The most common mitochondrial antigen in PBC is the M2 antigen.


39. A. Gilbert syndrome.
Rationale: Hyperbilirubinemia in Gilbert syndrome can be precipitated by dehydration, fasting, sepsis, and other causes of stress. However, the hyperbilirubinemia is mild and composed of unconjugated bilirubin.
B. Persistent intrahepatic cholestasis.
Rationale: Sepsis can be associated with intrahepatic cholestasis as illustrated in this patient during the acute phase. However, persistently elevated direct bilirubin in the absence of elevation in other markers of cholestasis, such as ALP and GGT, is unlikely to be caused by cholestasis.
C. Persistent hemolysis.
Rationale: Although intravascular hemolysis can rarely be associated with S. aureus sepsis, it will cause mild elevations of unconjugated bilirubin, not direct bilirubin.
D. Delayed clearing of albumin-bound bilirubin (delta fraction).
Rationale: In patients with prolonged cholestasis and elevated bilirubin, formation of a covalent complex of albumin and bilirubin (δ-bilirubin) can lead to persistent elevation of direct reacting bilirubin, whereas other bilirubin fractions quickly normalize after removal of the cholestasis. Although the unconjugated and glucuronide-conjugated bilirubin fractions have half-lives of minutes in circulation, δ-bilirubin has the half-life of albumin, about 2 to 3 weeks. In contrast to conjugated bilirubin, which can be excreted by the kidney and cause positive bilirubin urinalysis, δ-bilirubin, because of albumin conjugation, does not appear in the urine of patients with healthy renal glomeruli.
E. Heterophilic antibody interference with bilirubin assay.
Rationale: Bilirubin is assayed by a chemical method. There are no described instances of interference by antibodies.


Major points of discussion


 Intrahepatic cholestasis can accompany bacterial sepsis in 1% to 6% of adults and a slightly higher percentage of neonates. Gram-negative rods such as Escherichia coli and Klebsiella species are more common causes, whereas Staphylococcus and Streptococcus species sepsis can also cause jaundice. Congestion, hypoxia, bacterial products, and proinflammatory cytokines may be contributors to hepatic injury leading to cholestasis.


 Although usually mild, sepsis-associated jaundice can be intense, particularly in patients with preexisting hepatobiliary disease, and may divert attention from the diagnosis of underlying sepsis.


 Fractionation of bilirubin, for example, by liquid chromatography, shows the presence of four fractions:


 Unconjugated bilirubin, loosely bound to albumin (α-bilirubin)


 Bilirubin monoglucuronide (β-bilirubin)


 Bilirubin diglucuronide (γ-bilirubin)


 Bilirubin covalently conjugated to albumin (δ-bilirubin)


 Bilirubin is usually assayed by a colorimetric chemical method derived from the diazo reaction first described by Erlich in 1883. Most current methods use caffeine benzoate to displace unconjugated bilirubin from albumin and accelerate its reaction with a diazotized chromogen. In the presence of the accelerator, both conjugated and unconjugated bilirubin react quickly with the diazo dye, and the result is labeled total bilirubin.


 When the accelerator is omitted, unconjugated bilirubin reacts very slowly, whereas the other fractions react quickly but incompletely. The result is labeled as direct bilirubin. Unconjugated bilirubin can then be estimated by subtraction of total bilirubin minus direct bilirubin. However, accuracy of this estimation is hampered by the fact that between 1% and 10% of the unconjugated bilirubin can be measured as direct bilirubin.


 δ-Bilirubin is present in significant amounts in patients with persistent cholestasis and conjugated hyperbilirubinemia, probably as a result of displacement of glucuronide by albumin. Because δ-bilirubin can be variably measured as direct bilirubin, elevated δ-bilirubin can result in overestimation of the levels of conjugated bilirubin and underestimation of unconjugated bilirubin. Moreover, in patients recovering from cholestasis, persistent elevation of bilirubin can be misconstrued as failure to recover from the disease.


 Alternative methods for bilirubin determination using dry-slide chemistry can discriminate conjugated (β and γ-bilirubin (Bc) from unconjugated (α) bilirubin (Bu) by spectrophotometric scanning in the present of a cationic mordant. In this method, δ-bilirubin is not measured at all. When used in conjunction with a total bilirubin method, δ-bilirubin can be calculated as the difference of total bilirubin − Bc − Bu.12,38


40. A. Antinuclear antibodies (ANAs) with a diffuse pattern.
Rationale: ANAs can be seen in PBC. The pattern is usually a nuclear rim pattern, indicating autoantibodies against the nuclear envelope, or a nuclear dot pattern. Antibodies showing a nuclear rim pattern include antinuclear pore glycoprotein 210, present in about one fourth of patients with PBC, and nuclear protein p62, present in about 30% to 55% of the cases. Both gp210 and p62 autoantibodies are highly specific (> 97%) for PBC and good predictors of advanced disease, hepatic failure, and poor prognosis. Antibodies showing a nuclear dot pattern include sp100 proteins, promyelocytic leukemia protein, NDP52, and sp140 proteins. Of these, sp140 proteins are highly specific for PBC and present in about 15% of the cases.
B. Anti-SMA.
Rationale: Anti-SMAs, directed against antigens such as smooth muscle actin, myosin, tropomyosin, and troponin, can be present in PBC in up to 25% of cases but are typically associated with autoimmune hepatitis type III and may indicate the concurrent presence of this disease in the setting of PBC.
C. ANCA.
Rationale: ANCAs are primarily associated with systemic vasculitides but can be present in a minority (2% to 26%) of PBC patients.
D. Anti–pyruvate dehydrogenase (PDC-M2) antibodies.
Rationale: Antimitochondrial antibodies directed against the pyruvate dehydrogenase complex (labeled as M2 antibodies) are the predominant antibodies in PBC, present in 90% to 95% of the cases.
E. Anti–glutamic acid decarboxylase antibodies (GADA).
Rationale: Anti-GADAs are associated with type 1 diabetes and rarely (< 5%) are present in PBC.


Major points of discussion


 PBC is an autoimmune disorder characterized by progressive destruction of small and medium-sized intrahepatic bile ducts, portal inflammation, and progressive liver fibrosis leading to cirrhosis and hepatic failure.


 PBC primarily affects middle-aged women and can present with cholestasis (e.g., jaundice, pruritus, xanthomas), hepatomegaly, and cirrhosis with portal hypertension, ascites, and gastrointestinal bleeding. PBC is often associated with other autoimmune disorders such as Sjögren syndrome, arthropathy, CREST syndrome (calcinosis, Raynaud’s phenomenon, esophageal dysmotility, sclerodactyly, and telangiectasia), and autoimmune thyroiditis.


 In developed countries, most cases are asymptomatic at presentation and detected by abnormal LFT results. Most commonly, markers of cholestasis, such as elevation of ALP and GGT, are seen. Less commonly, the patient presents with jaundice and conjugated hyperbilirubinemia. When the disease progresses to marked liver fibrosis and cirrhosis, markers of hepatocyte damage, such as mildly elevated aminotransferases, can be detected. With the development of hepatic insufficiency, hypoalbuminemia, prolonged prothrombin time, and elevated conjugated and unconjugated bilirubin can be seen. Hyperbilirubinemia is a sign of advanced or severe disease and an indicator for the possibility of liver transplantation.


 Antimitochondrial antibodies (M2, M4, M8, M9) are present in all patients with PBC and remain a pathognomonic hallmark of the diagnosis. Anti-M2 antibodies are directed against the pyruvate decarboxylase complex (PDC) in the inner mitochondrial membrane, and M4, M8, and M9 bind to the outer membrane. In 90% to 95% of cases, autoantibodies directed against the lipoyl domain of the E2 subunit of mitochondrial PDC are present. Anti-M2 antibodies can also be directed against other members of the PDC complex, including the E2 subunit of the branched-chain 2-oxoacid dehydrogenase complex, the E2 subunit of the 2-oxoglutarate dehydrogenase complex, E1t α-subunits of PDC, and E3-binding protein (protein X). Other mitochondrial targets of autoantibodies present in PBC include the β- and γ-subunits of F1F0 adenosine triphosphatase and sulfite oxidase (M4).


 Although antimitochondrial antibodies are very sensitive for the diagnosis of PBC, they are not entirely specific. First, they can present many years in advance of significant hepatic disease, although in this case they can serve as a predictor for development of PBC. Second, they can be present in other autoimmune disorders, including Sjögren syndrome, scleroderma, autoimmune hepatitis, and infectious diseases such as viral hepatitis and tuberculosis.


 The pathogenesis of PBC implicates liver-infiltrating autoreactive T cells against the dominant PDC-E2 autoantigen. Levels of antimitochondrial antibodies reflect the B-cell response and do not correlate with the severity of disease.


 Antinuclear antibodies, directed against pore glycoprotein 210, nuclear protein p62, sp100 proteins, promyelocytic leukemia protein, NDP52, and sp140 proteins, can be positive in about 50% of the minority of patients (5% to 10%) without detectable antimitochondrial antibodies.14,17,24


41. A. No reflex test is needed.
Rationale: A positive result on the anti-HAV total assay indicates the presence of antibodies, and it should be determined whether they are immunoglobulin M (IgM) (as in acute infection) or immunoglobulin G (IgG) (as in chronic/past infection).
B. Hepatitis A IgM testing.
Rationale: Anti-HAV IgM indicates an acute infection and is the appropriate reflex test if an assay for total anti-HAV antibodies is used as the initial screen for HAV. If the anti-HAV IgM test result is negative, one can assume that anti-HAV IgG is positive, which is indicative of a past/chronic infection.
C. Hepatitis A IgA testing.
Rationale: There is not a routinely used or available test.
D. Hepatitis D IgG testing.
Rationale: Hepatitis D is associated with hepatitis B infection, not hepatitis A.
E. Hepatitis A nucleic acid testing.
Rationale: Nucleic acid testing for HAV is not approved by the U.S. Food and Drug Administration (FDA) for diagnostic purposes.


Major points of discussion


 There are three serologic assays for anti-HAV antibodies available:


Total antibodies to HAV


Only immunoglobulin G (IgG) antibodies to HAV


Only immunoglobulin M (IgM) antibodies to HAV


A combination of all or some of these three assays can be used for the laboratory workup of HAV infection.


 In the laboratory diagnosis of HAV infection, one possible algorithm uses a two-step procedure. The first step entails testing for the presence of total (IgG + IgM) anti-HAV antibodies; and if positive, the second step entails testing for the presence of IgM anti-HAV antibodies (indicating acute infection). If the IgM testing is negative, it is assumed that the patient’s positive result on the screening test was caused by a chronic or past infection.


 A second possible method for testing uses two separate tests for IgG and for IgM. If this method is used, the test for IgG assesses chronic/past infection, whereas the IgM-based assay is used to assess acute infection.


 HAV is an RNA-based virus. It is transmitted through the fecal-oral route, usually through contaminated food or water or by direct contact with an infected individual. The incubation time is 2 to 6 weeks.


 The diagnosis and detection of HAV infection are routinely performed in the clinical laboratory using only serologic assays, not molecular assays. A reverse-transcriptase polymerase chain reaction (RT-PCR)-based approach has been successfully demonstrated in the scientific literature, but is not used in routine clinical practice. This is not the case for HBV and HCV, for which molecular assays are important in the management of infected patients.


42. A. Hepatitis B surface antigen.
Rationale: Hepatitis B surface antigen is the first-line diagnostic test for active HBV infection.
B. Anti–hepatitis B core total antibodies (IgG + IgM).
Rationale: This test indicates current or prior infection.
C. Anti–hepatitis B surface antibodies.
Rationale: This test indicates immunity after infection or vaccination.
D. Anti–hepatitis Be antibody.
Rationale: This test can be used to assess treatment response.
E. Hepatitis B genotyping.
Rationale: Genotyping of the hepatitis viruses is useful for epidemiologic and prognostic purposes.


Major points of discussion


 Hepatitis B virus (HBV) is a DNA-based virus. It is transmitted through contact with blood, semen, and other body fluids. The incubation period is 45 to 160 days.


 The appropriate serologic tests for hepatitis B infection include (1) hepatitis B surface antigen, (2) anti–hepatitis B core protein antibodies, (3) total anti-HBV antibodies, (4) anti–hepatitis B envelope protein antibodies, and (5) anti–hepatitis B core protein immunoglobulin M (IgM) antibodies.


 The first-line diagnostic for acute infection is the detection of hepatitis B surface antigen. A second test useful for assessing acute infection is IgM-based testing for anti–hepatitis B core.


 Several molecular tests are available for hepatitis B. These include tests used for monitoring response, predicting response, detecting mutation, and genotyping for epidemiologic and prognostic purposes.


 Hepatitis D infection is usually associated with hepatitis B infection. Coinfection of the two viruses can result in an unusually aggressive disease characterized by fulminant hepatic failure.


43. A. Recombinant immunoblot assay (RIBA) test for HCV.
Rationale: This is the confirmation test that should be performed after an initial positive screening.
B. HCV genotyping.
Rationale: This test is used to predict the likelihood of a therapeutic response.
C. Anti-HCV immunoglobulin M (IgM) antibodies.
Rationale: This test is not clinically used.
D. Anti-HCV immunoglobulin G (IgG) antibodies.
Rationale: This is the first-line screening test for acute and chronic HCV infection.
E. Hepatitis C surface antigen.
Rationale: There is no such antigen or test available.


Major points of discussion


 HCV is an RNA-based virus that is transmitted primarily through intravenous drug use (in the developed world) and through blood transfusions and unsafe medical procedures (in the developing world). The incubation period has a wide range of 14 to 180 days.


 The first-line screening test for HCV includes anti-HCV antibody testing.


 The Centers for Disease Control and Prevention (CDC) recommend one-time testing for HCV in anyone born between 1945 and 1965, using an HCV antibody test. Those with positive test results should be tested with a molecular test to determine whether an active infection is present.


 In HCV infection, elevated ALT levels, along with positive viral load, are sufficient for treatment initiation.


 Molecular assays for HCV include genotyping assays that are useful for predicting response and/or duration of treatment, and viral load assays. The latter are useful for highly sensitive quantitation, can confirm active infection, can predict response, and can be used for monitoring.

Stay updated, free articles. Join our Telegram channel

Dec 23, 2016 | Posted by in PATHOLOGY & LABORATORY MEDICINE | Comments Off on Clinical Chemistry: Liver, Gastrointestinal, Pancreas, Biliary Tract

Full access? Get Clinical Tree

Get Clinical Tree app for offline access